Chapter 40: Management of Patients with Gastric and Duodenal Disorders

Lakukan tugas rumah & ujian kamu dengan baik sekarang menggunakan Quizwiz!

Which of the following is considered an early symptom of gastric cancer?

Pain relieved by antacids

Which medication classification represents a proton (gastric acid) pump inhibitor? Sucralfate Omeprazole Metronidazole Famotidine

Omeprazole Explanation: Omeprazole decreases gastric acid by slowing the hydrogen-potassium adenosine triphosphatase pump on the surface of the parietal cells. Sucralfate is a cytoprotective drug. Famotidine is a histamine-2 receptor antagonist. Metronidazole is an antibiotic, specifically an amebicide.

A client is recovering from gastric surgery. Toward what goal should the nurse progress the client's enteral intake?

Six small meals daily with 120 mL fluid between meals

A client is recovering from gastric surgery. Toward what goal should the nurse progress the client's enteral intake?

Six small meals daily with 120 mL fluid between meals After the return of bowel sounds and removal of the nasogastric tube, the nurse may give fluids, followed by food in small portions. Foods are gradually added until the client can eat six small meals a day and drink 120 mL of fluid between meals.

When caring for a client with an acute exacerbation of a peptic ulcer, the nurse finds the client doubled up in bed with severe pain in the right shoulder. What is the intial appropriate action by the nurse?

Assess the client's abdomen and vital signs.

Morbid obesity is defined as being how many pounds over the person's ideal body weight?

100

A nursing student is preparing a teaching plan about peptic ulcer disease. The student knows to include teaching about the percentage of clients with peptic ulcers who experience bleeding. The percentage is

15%

The nurse determines that teaching for the client with peptic ulcer disease has been effective when the client states:

"I have learned some relaxation strategies that decrease my stress."

A nurse is performing discharge teaching with a client who had a total gastrectomy. Which statement indicates the need for further teaching?

"I will have to take vitamin B12 shots up to 1 year after surgery."

The nurse is caring for a client who has been experiencing symptoms of acute gastritis. The client is now reporting an absence of symptoms and increase in hunger. Which action will the nurse take?

Provide applesauce and toast.

The nurse advises the patient who has just been diagnosed with acute gastritis to:

Refrain from food until the GI symptoms subside.

Which of the following is the most successful treatment for gastric cancer?

Removal of the tumor

Which of the following clients is at highest risk for peptic ulcer disease?

Client with blood type O Clients with blood type O are more susceptible to peptic ulcers than those with blood types A, B, and AB.

A client comes to the clinic after developing a headache, abdominal pain, nausea, hiccuping, and fatigue about 2 hours ago. The client tells the nurse that the last food was buffalo chicken wings and beer. Which medical condition does the nurse find to be most consistent with the client's presenting problems?

Acute gastritis

Which of the following clients is at highest risk for peptic ulcer disease? Client with blood type A Client with blood type O Client with blood type B Client with blood type AB

Client with blood type O Explanation: Clients with blood type O are more susceptible to peptic ulcers than those with blood types A, B, and AB.

A client has given a confirmed diagnosis of gastric cancer. Two more procedures may be performed to assess tumor depth and lymph node involvement and surgical resectability. Which two are the procedures? Choose the two that apply.

Computed tomography (CT) Endoscopic ultrasound Esophagogastroduodenoscopy for biopsy and cytologic washings is the diagnostic study of choice, and a barium x-ray examination of the upper GI tract may also be performed. Endoscopic ultrasound is an important tool to assess tumor depth and any lymph node involvement. Pelvic ultrasound is not used to confirm the diagnosis of gastric cancer.

A morbidly obese client asks the nurse if medications are available to assist with weight loss. The nurse knows that the client would not be a candidate for phentermine if the following is part of the client's health history:

Coronary artery disease Phentermine, which requires a prescription, stimulates central noradrenergic receptors, causing appetite suppression. It may increase blood pressure and should not be taken by people with a history of heart disease, uncontrolled hypertension, hyperthyroidism, or glaucoma.

A client admitted for treatment of a gastric ulcer is being prepared for discharge. The client will follow a regimen of antacid therapy. Discharge teaching should include which instructions? Choose all that apply. - "Continue to take antacids even if your symptoms subside." - "You may take antacids with other medications." - "You may be prescribed H2-receptor antagonists for up to 1 year." - "Be sure to take antacids with meals." - "The antacids will make you sleepy, so do not operate machinery while taking them."

Correct response: "Continue to take antacids even if your symptoms subside." "You may be prescribed H2-receptor antagonists for up to 1 year." Explanation: The client is advised to adhere to and complete the medication regimen to ensure complete healing of the ulcer. Because most clients become symptom-free within 1 week, the nurse stresses the importance of following the prescribed regimen so that the healing process can continue uninterrupted and the return of chronic ulcer symptoms can be prevented. Maintenance dosages of H2-receptor antagonists are usually recommended for 1 year. Taking antacids concomitantly with other drugs should be avoided. For best results antacids should be taken 1 hour before or 2 hours after meals. Antacids will not make the client sleepy.

The nurse determines that teaching for the client with peptic ulcer disease has been effective when the client makes which statement? - "I should stop all my medications if I develop any side effects." - "I should continue my treatment regimen as long as I have pain." - "I have learned some relaxation strategies that decrease my stress." - "I can buy whatever antacids are on sale because they all have the same effect."

Correct response: "I have learned some relaxation strategies that decrease my stress." Explanation: The nurse assists the client to identify stressful or exhausting situations. A hectic lifestyle and an irregular schedule may aggravate symptoms and interfere with regular meals taken in relaxed settings along with the regular administration of medications. The client may benefit from regular rest periods during the day, at least during the acute phase of the disease. Biofeedback, hypnosis, behavior modification, massage, or acupuncture may be helpful.

A nurse is teaching a client who has experienced an episode of acute gastritis. The nurse knows further education is necessary when the client makes which statement? - "I should feel better in about 24 to 36 hours." - "My appetite should come back tomorrow." - "I should limit alcohol intake, at least until symptoms subside." - "Once I can eat again, I should stick with bland foods."

Correct response: "My appetite should come back tomorrow." Explanation: The gastric mucosa is capable of repairing itself after an episode of gastritis. As a rule, the client recovers in about 1 day, although the appetite may be diminished for an additional 2 or 3 days. Acute gastritis is also managed by instructing the client to refrain from alcohol and food until symptoms subside. When the client can take nourishment by mouth, a nonirritating diet is recommended.

The nurse is caring for a patient who has been diagnosed with gastritis. To promote fluid balance when treating gastritis, the nurse knows that what minimal daily intake of fluids is required? - 1.0 L - 1.5 L - 2.0 L - 2.5 L

Correct response: 1.5 L Explanation: Daily fluid intake and output are monitored to detect early signs of dehydration (minimal fluid intake of 1.5 L/day, minimal output of 0.5 mL/kg/h).

A client who is being treated for pyloric obstruction has a nasogastric (NG) tube in place to decompress the stomach. The nurse routinely checks for obstruction which would be indicated by what amount? - 150 mL - 250 mL - 350 mL - 450 mL

Correct response: 450 mL Explanation: A residual of greater than 400 mL strongly suggests obstruction.

The nurse reviews dietary guidelines with a client who had a gastric banding. Which teaching points are included? Select all that apply. - Eat six meals a day. - Limit meal size to 450 to 500 mL. - Do not eat and drink at the same time. - Drink plenty of water, from 90 minutes after each meal to 15 minutes before each meal. - avoid fruit drinks and soda.

Correct response: Do not eat and drink at the same time. Drink plenty of water, from 90 minutes after each meal to 15 minutes before each meal. Avoid fruit drinks and soda. Explanation: Total meal size should be restricted to less than 8 oz or 240 mL. Three meals a day are recommended.

A client seeking care because of recurrent heartburn and regurgitation is subsequently diagnosed with a hiatal hernia. Which of the following should the nurse include in health education?

"Instead of eating three meals a day, try eating smaller amounts more often."

When caring for a client with an acute exacerbation of a peptic ulcer, the nurse finds the client doubled up in bed with severe pain to his right shoulder. The intial appropriate action by the nurse is to

Assess the client's abdomen and vital signs.

As a nurse completes the admission assessment of a client admitted for gastric bypass surgery, the client states, "Finally! I'll be thin and able to eat without much concern." How should the nurse intervene?

Evaluate the client's understanding of the procedure.

A nurse has obtained an order to remove a client's NG tube that was placed for feeding. What is the nurse's best initial action?

Explain the process clearly to the client.

A client has a new order for metoclopramide. What potential side effects should the nurse educate the client about? Extrapyramidal Peptic ulcer disease Decreased gastric motility nausea

Extrapyramidal

Which medication is classified as a histamine-2 receptor antagonist?

Famotidine

Which medication is classified as a histamine-2 receptor antagonist? Famotidine Esomeprazole Lansoprazole Metronidazole

Famotidine Explanation: Famotidine is a histamine-2 receptor antagonist. Lansoprazole and esomeprazole are proton pump inhibitors (PPIs). Metronidazole is an antibiotic.

A client is prescribed a histamine (H2)-receptor antagonist. The nurse understands that this might include which medication(s)? Select all that apply.

Famotidine Cimetidine Nizatidine

A client has been taking famotidine at home. What teaching should the nurse include with the client?

Famotidine will inhibit gastric acid secretions.

A client has been taking famotidine at home. What teaching should the nurse include with the client? Famotidine will inhibit gastric acid secretions. Famotidine will neutralize acid in the stomach. Famotidine will shorten the time required for digestion in the stomach. Famotidine will improve the mixing of foods and gastric secretions.

Famotidine will inhibit gastric acid secretions. Explanation: Famotidine is useful for treating and preventing ulcers and managing gastroesophageal reflux disease. It functions by inhibiting the action of histamine at the H-2 receptor site located in the gastric parietal cells, thus inhibiting gastric acid secretion. Famotidine will not neutralize acid in the stomach, but inhibits acid secretion. Famotidine will not shorten digestion time and will not improve food mixing with gastric secretions.

The nurse assesses a patient who recently had a nasoenteric intubation. Symptoms of oliguria, lethargy, and tachycardia in the patient would indicate to the nurse what common complication?

Fluid volume deficit

A client with gastric cancer is planning to receive chemotherapy. Which medication will be used as the primary agent to improve the tumor response rate? Paclitaxel Carboplatin Docetaxel Fluorouracil

Fluorouracil Explanation: In instances where the gastric tumor is not resectable, treatment with chemotherapy may offer further control of the disease or palliation. Chemotherapy may also be used in addition to surgery as adjuvant treatment of gastric cancer. For improved tumor response rates, it is more common to administer combination chemotherapy, primarily fluorouracil-based therapy, with other agents. Paclitaxel, docetaxel, and carboplatin are chemotherapeutic agents used to treat gastric cancer, however, are not used in combination in order to improve tumor response rates.

A client is recovering from gastric surgery. Which of the following is the correct position for the nurse to place this client?

Fowler's

A client has a new order for metoclopramide. The nurse identifies that this medication can be safely administered for which condition? Gastroesophageal reflux disease Peptic ulcer with melena Diverticulitis with perforation Gastritis

Gastroesophageal reflux disease

A client with human immunodeficiency virus (HIV) comes to the clinic and is experiencing white patches on the lateral border of the tongue. What type of lesions does the nurse document?

Hairy leukoplakia

The nurse is assessing a client with an ulcer for signs and symptoms of hemorrhage. The nurse interprets which condition as a sign/symptom of possible hemorrhage?

Hematemesis

A client with Crohn's disease is losing weight. For which reason will the nurse anticipate the client being prescribed parenteral nutrition?

Impaired ability to absorb food

Which of the following dietary guidelines should be followed following bariatric surgery? Select all that apply.

Include two protein snacks per day. Eat slowly. Eat three meals per day.

A patient who had bariatric surgery complains of diarrhea. The nurse recommends which of the following dietary changes?

Increased fiber The nurse recommends that the patient increase fiber in the diet because a high-fiber diet can decrease both diarrhea and constipation after bariatric surgery.

A client is being evaluated for esophageal cancer. What initial manifestation of esophageal cancer should the nurse assess?

Increasing difficulty in swallowing

A medical nurse who is caring for a client being discharged home after a radical neck dissection has collaborated with the home health nurse to develop a plan of care for this client. What is a priority psychosocial outcome for this client?

Indicates acceptance of altered appearance and demonstrates positive self-image

The nurse is reviewing the medication record of a client with acute gastritis. Which medication, if noted on the client's record, would the nurse question?

Indomethacin (Indocin) Indomethacin is a nonsterioidal anti-inflammatory drug and can cause ulceration of the esophagus, stomach, or small intestine. Indomethacin is contraindicated in a client with a gastrointestinal disorder. The other medications are not contraindicated in clients with gastrointestinal disorders.

A nurse is providing care for a client whose neck dissection surgery involved the use of a graft. When assessing the graft, the nurse should prioritize data related to what nursing diagnosis?

Ineffective tissue perfusion

A client presents to the clinic reporting vomiting and burning in her mid-epigastria. The nurse knows that in the process of confirming peptic ulcer disease, the health care provider is likely to order a diagnostic test to detect the presence of what? Infection with Helicobacter pylori Excessive stomach acid secretion An incompetent pyloric sphincter A metabolic acid-base imbalance

Infection with Helicobacter pylori

A client presents to the clinic reporting vomiting and burning in the mid-epigastria. The nurse knows that in the process of confirming peptic ulcer disease, the health care provider is likely to order a diagnostic test to detect the presence of what?

Infection with Helicobacter pylori

A nursing student is caring for a client with gastritis. Which of the following would the student recognize as a common cause of gastritis? Choose all that apply.

Ingestion of strong acids Irritating foods Overuse of aspirin

A nursing student is caring for a client with gastritis. Which of the following would the student recognize as a common cause of gastritis? Choose all that apply.

Ingestion of strong acids Irritating foods Overuse of aspirin

A nursing student is caring for a client with gastritis. Which of the following would the student recognize as a common cause of gastritis? Choose all that apply. Participation in highly competitive sports Irritating foods Ingestion of strong acids DASH diet Overuse of aspirin

Ingestion of strong acids Irritating foods Overuse of aspirin Explanation: Acute gastritis is often caused by dietary indiscretion-a person eats food that is irritating, too highly seasoned, or contaminated with disease-causing microorganisms. Other causes of acute gastritis include overuse of aspirin and other nonsteroidal anti-inflammatory drugs (NSAIDs), excessive alcohol intake, bile reflux, and radiation therapy. A more severe form of acute gastritis is caused by the ingestion of strong acid or alkali, which may cause the mucosa to become gangrenous or to perforate. A DASH diet is an acronym for Dietary Approaches to Stop Hypertension, which would not cause gastritis. Participation in competitive sports also would not cause gastritis.

The client has been taking famotidine (Pepcid) at home. The nurse prepares a teaching plan for the client indicating that the medication acts primarily to achieve which of the following?

Inhibit gastric acid secretions. Famotidine is useful for treating and preventing ulcers and managing gastroesophageal reflux disease. It functions by inhibiting the action of histamine at the H-2 receptor site located in the gastric parietal cells, thus inhibiting gastric acid secretion.

A nurse is preparing to place a client's prescribed nasogastric tube. What anticipatory guidance should the nurse provide to the client?

Insertion is likely to cause some gagging.

A client has been admitted to the hospital after diagnostic imaging revealed the presence of a gastric outlet obstruction (GOO). What is the nurse's priority intervention?

Insertion of an NG tube for decompression

A client with an H. pylori infection asks why bismuth subsalicylate is prescribed. Which response will the nurse make?

"It aids in the healing of the stomach lining."

A nurse is teaching a client who has experienced an episode of acute gastritis and knows further education is necessary when the client makes the following statement:

"My appetite should come back tomorrow."

A client comes to the clinic reporting pain in the epigastric region. What statement by the client suggests the presence of a duodenal ulcer? "I seem to have bowel movements more often than I usually do." "I know that my father and my grandfather both had ulcers." "The pain really interferes with my quality of life." "My pain resolves when I have something to eat."

"My pain resolves when I have something to eat."

A client with a peptic ulcer is diagnosed with Helicobacter pylori infection. The nurse is teaching the client about the medications prescribed, including metronidazole, omeprazole, and clarithromycin. Which statement by the client indicates the best understanding of the medication regimen? "My ulcer will heal because these medications will kill the bacteria." "I should take these medications only when I have pain from my ulcer." "These medications will coat the ulcer and decrease the acid production in my stomach." "The medications will kill the bacteria and stop the acid production."

"The medications will kill the bacteria and stop the acid production." Explanation: Currently, the most commonly used therapy for peptic ulcers is a combination of antibiotics, proton-pump inhibitors, and bismuth salts that suppress or eradicate H. pylori. Recommended therapy for 10 to 14 days includes triple therapy with two antibiotics (e.g., metronidazole [Flagyl] or amoxicillin [Amoxil] and clarithromycin [Biaxin]) plus a proton-pump inhibitor (e.g., lansoprazole [Prevacid], omeprazole [Prilosec], or rabeprazole [Aciphex]), or quadruple therapy with two antibiotics (metronidazole and tetracycline) plus a proton-pump inhibitor and bismuth salts (Pepto-Bismol). Research is being conducted to develop a vaccine against H. pylori.

A client being treated for a peptic ulcer seeks medical attention for vomiting blood. Which statement indicates to the nurse the reason for the client developing hematemesis? "I felt better but then just got really nauseated and threw up." "I think the soda that I drank irritated my stomach." "The pain stopped so I stopped taking the medications." "I only ate dinner yesterday and it gave me an upset stomach."

"The pain stopped so I stopped taking the medications." Explanation: The client should be instructed to adhere to and complete the medication regimen to ensure complete healing of the peptic ulcer. Because most clients become symptom free within a week, it should be stressed to the client the importance of following the prescribed regimen so that the healing process can continue uninterrupted and the return of symptoms can be prevented. Since the client stopped taking the medication, the ulcer was not healed and became worse. The statements about soda, being nauseated, and eating only one meal would not explain the reason for the client's new onset of hematemesis during treatment for a peptic ulcer.

A patient with a diagnosis of peptic ulcer disease has just been prescribed omeprazole (Prilosec). How should the nurse best describe this medication's therapeutic action?

"This medication will reduce the amount of acid secreted in your stomach."

A nurse is providing preprocedure education for a client who will undergo a lower GI tract study the following week. What should the nurse teach the client about bowel preparation? "For 24 hours before the test, insert a glycerin suppository every 4 hours." "You'll need to have enemas the day before the test." "Starting today, take over-the-counter stool softeners twice daily." "You'll need to fast for at least 18 hours prior to your test."

"You'll need to have enemas the day before the test."

A client is prescribed tetracycline to treat peptic ulcer disease. Which of the following instructions would the nurse give the client? -"Take the medication with milk." -"Be sure to wear sunscreen while taking this medicine." -"Expect a metallic taste when taking this medicine, which is normal." -"Do not drive when taking this medication."

-"Be sure to wear sunscreen while taking this medicine." Tetracycline may cause a photosensitivity reaction in clients. The nurse should caution the client to use sunscreen when taking this drug. Dairy products can reduce the effectiveness of tetracycline, so the nurse should not advise him or her to take the medication with milk. A metallic taste accompanies administration of metronidazole (Flagyl). Administration of tetracycline does not necessitate driving restrictions.

A nurse is teaching a client with gastritis about the need to avoid the intake of caffeinated beverages. The client asks why this is so important. Which of the following explanations from the nurse would be most accurate? -"Caffeine stimulates the central nervous system and thus gastric activity and secretions, which need to be minimized to promote recovery." -"Caffeine increases the fluid volume in your system, which irritates your digestive organs." -"Caffeine intake can cause tears in your esophagus and intestines, which can lead to hemorrhage." -"Caffeine can interfere with absorption of vitamin B12, which leads to anemia and further digestive problems."

-"Caffeine stimulates the central nervous system and thus gastric activity and secretions, which need to be minimized to promote recovery." Caffeine is a central nervous system stimulant that increases gastric activity and pepsin secretion. Caffeine is a diuretic that causes decreased fluid volume and potential dehydration. It does not lead to hemorrhage and does not interfere with absorption of vitamin B12.

A client with gastric cancer is scheduled to undergo a Billroth II procedure. The client's spouse asks how much of the client's stomach will be removed. Which of the following would be the most accurate response from the nurse? -Approximately 25% -Approximately 50% -Approximately 75% -The amount will depend on the client's weight.

-Approximately 75% The Billroth II is a wide resection that involves removing approximately 75% of the stomach and decreases the possibility of lymph node spread or metastatic recurrence.

The nurse determines that teaching for the client with peptic ulcer disease has been effective when the client states: -"I should stop all my medications if I develop any side effects." -"I should continue my treatment regimen as long as I have pain." -"I have learned some relaxation strategies that decrease my stress." -"I can buy whatever antacids are on sale because they all have the same effect."

-"I have learned some relaxation strategies that decrease my stress." The nurse assists the client to identify stressful or exhausting situations. A hectic lifestyle and an irregular schedule may aggravate symptoms and interfere with regular meals taken in relaxed settings along with the regular administration of medications. The client may benefit from regular rest periods during the day, at least during the acute phase of the disease. Biofeedback, hypnosis, behavior modification, massage, or acupuncture may be helpful.

After teaching a client who has had a Roux-en-Y gastric bypass, which client statement indicates the need for additional teaching? -"I need to chew my food slowly and thoroughly." -"I need to drink 8 ounces of water before eating." -"A total serving should amount to be less than one cup." -"I should pick cereals with less than 2 g of fiber per serving."

-"I need to drink 8 ounces of water before eating." After a Roux-en-Y gastric bypass, the client should not drink fluids with meals, withholding fluids for 15 minutes before eating to 90 minutes after eating. Chewing foods slowly and thoroughly, keeping total serving sizes to less than 1 cup, and choosing foods such as breads, cereals, and grains that provide less than 2 g of fiber per serving.

A nurse is performing discharge teaching with a client who had a total gastrectomy. Which statement indicates the need for further teaching? -"I'm going to visit my pastor weekly for a while." -"I will have to take vitamin B12 shots up to 1 year after surgery." -"I will call my physician if I begin to have abdominal pain." -"I will weight myself each day and record the weight."

-"I will have to take vitamin B12 shots up to 1 year after surgery." After a total gastrectomy, a client will need to take vitamin B12 shots for life. Dietary B12 is absorbed in the stomach, and the inability to absorb it could lead to pernicious anemia. Visiting clergy for emotional support is normal after receiving a cancer diagnosis. This action should be encouraged by the nurse. It's appropriate for the client to call the physician if he experiences signs and symptoms of intestinal blockage or obstruction, such as abdominal pain. Because a client with a total gastrectomy will receive enteral feedings or parenteral feedings, he should weigh himself each day and keep a record of the weights.

A nurse is teaching a client who has experienced an episode of acute gastritis and knows further education is necessary when the client makes the following statement: -"I should feel better in about 24 to 36 hours." -"My appetite should come back tomorrow." -"I should limit alcohol intake, at least until symptoms subside." -"Once I can eat again, I should stick with bland foods."

-"My appetite should come back tomorrow." The gastric mucosa is capable of repairing itself after an episode of gastritis. As a rule, the client recovers in about 1 day, although the appetite may be diminished for an additional 2 or 3 days. Acute gastritis is also managed by instructing the client to refrain from alcohol and food until symptoms subside. When the client can take nourishment by mouth, a nonirritating diet is recommended.

A client with a peptic ulcer is diagnosed with Helicobacter pylori infection. The nurse is teaching the client about the medications prescribed, including metronidazole (Flagyl), omeprazole (Prilosec), and clarithromycin (Biaxin). Which statement by the client indicates the best understanding of the medication regimen? -"My ulcer will heal because these medications will kill the bacteria." -"I should take these medications only when I have pain from my ulcer." -"The medications will kill the bacteria and stop the acid production." -"These medications will coat the ulcer and decrease the acid producation in my stomach."

-"The medications will kill the bacteria and stop the acid production." Currently, the most commonly used therapy for peptic ulcers is a combination of antibiotics, proton-pump inhibitors, and bismuth salts that suppress or eradicate H. pylori. Recommended therapy for 10 to 14 days includes triple therapy with two antibiotics (eg, metronidazole [Flagyl] or amoxicillin [Amoxil] and clarithromycin [Biaxin]) plus a proton-pump inhibitor (eg, lansoprazole [Prevacid], omeprazole [Prilosec], or rabeprazole [Aciphex]), or quadruple therapy with two antibiotics (metronidazole and tetracycline) plus a proton-pump inhibitor and bismuth salts (Pepto-Bismol). Research is being conducted to develop a vaccine against H. pylori.

The nurse is caring for a patient who has been diagnosed with gastritis. To promote fluid balance when treating gastritis, the nurse knows that what minimal daily intake of fluids is required? -1.0 L -1.5 L -2.0 L -2.5 L

-1.5 L Daily fluid intake and output are monitored to detect early signs of dehydration (minimal fluid intake of 1.5 L/day, minimal output of 0.5 mL/kg/h).

Morbid obesity is defined as being how many pounds over the person's ideal body weight? -100 -90 -75 -50

-100 Morbid obesity is a term applied to people who are more than two times their ideal body weight or whose body weight index (BMI) exceeds 30 kg/m2. Another definition of morbid obesity is body weight that is more than 100 pounds greater than the ideal body weight.

A nursing student is preparing a teaching plan about peptic ulcer disease. The student knows to include teaching about the percentage of clients with peptic ulcers who experience bleeding. The percentage is -Less than 5% -15% -25% -Greater than 50%

-15% Fifteen percent of clients with peptic ulcer experience bleeding.

A client weighs 215 lbs and is 5' 8" tall. The nurse would calculate this client's body mass index (BMI) as which of the following? -19.5 -24.8 -32.7 -44.9

-32.7 Using the formula for BMI, the client's weight in pounds (215) is divided by the height in inches squared (68 inches squared) and then multiplied by 703. The result would be 32.7.

The nurse is caring for a client recovering from open reduction, internal fixation (ORIF) of the mandible. Which teachings will the nurse provide to the client after the surgery? Select all that apply.

-Abstain from smoking -Refrain from ingesting alcohol -Use medicated oral rinses as prescribed -Follow a liquid or soft diet for 4 to 6 weeks

A client reports to the clinic, stating that she rapidly developed headache, abdominal pain, nausea, hiccuping, and fatigue about 2 hours ago. For dinner, she ate buffalo chicken wings and beer. Which of the following medical conditions is most consistent with the client's presenting problems? -Acute gastritis -Duodenal ulcer -Gastric cancer -Gastric ulcer

-Acute gastritis The client with acute gastritis may have a rapid onset of symptoms, including abdominal discomfort, headache, lassitude, nausea, anorexia, vomiting, and hiccuping, which can last from a few hours to a few days. Acute gastritis is often caused by dietary indiscretion—a person eats food that is irritating, too highly seasoned, or contaminated with disease-causing microorganisms.

A client has a family history of stomach cancer. Which of the following factors would further increase the client's risk for developing gastric cancer? Select all that apply. -High intake of fruits and vegetables -Age 55 years -Female gender -Caucasian ancestry -Previous infection with H. pylori

-Age 55 years -Previous infection with H. pylori The typical client with gastric cancer is between 40 and 70 years, but gastric cancer can occur in younger people. Men have a higher incidence of gastric cancer than women. Native Americans, Hispanic Americans, and African Americans are twice as likely as Caucasian Americans to develop gastric cancer. A diet high in smoked, salted, or pickled foods and low in fruits and vegetables may increase the risk of gastric cancer. Other factors related to the incidence of gastric cancer include chronic inflammation of the stomach, H. pylori infection, pernicious anemia, smoking, achlorhydria, gastric ulcers, subtotal gastrectomy more than 20 years ago, and genetics.

The nurse is assessing an 80-year-old client for signs and symptoms of gastric cancer. The nurse differentiates which as a sign/symptom of gastric cancer in the geriatric client, but not in a client under the age of 75? -Abdominal mass -Agitation -Hepatomegalia -Ascites

-Agitation The nurse understands that agitation, along with confusion and restlessness, may be the only signs/symptoms seen of gastric cancer in the older client. Abdominal mass, hepatomegaly, and ascites may all be signs/symptoms of advanced gastric cancer.

The nurse practitioner suspects that a patient may have a gastric ulcer after completing a history and physical exam. Select an indicator that can be used to help establish the distinction. -Amount of hydrochloric acid (HCL) secretion in the stomach -Sensitivity to the use of nonsteroidal anti-inflammatory drugs (NSAIDs) -Presence of H. pylori -Patient's age

-Amount of hydrochloric acid (HCL) secretion in the stomach A duodenal ulcer is characterized by hypersecretion of stomach acid, whereas a gastric ulcer evidences hyposecretion of stomach acid. The other three choices have similar characteristics in both types of ulcers.

When caring for a client with an acute exacerbation of a peptic ulcer, the nurse finds the client doubled up in bed with severe pain to his right shoulder. The intial appropriate action by the nurse is to -Notify the health care provider. -Irrigate the client's NG tube. -Place the client in the high-Fowler's position. -Assess the client's abdomen and vital signs.

-Assess the client's abdomen and vital signs. Signs and symptoms of perforation includes sudden, severe upper abdominal pain (persisting and increasing in intensity); pain may be referred to the shoulders, especially the right shoulder, because of irritation of the phrenic nerve in the diaphragm. The nurse should assess the vital signs and abdomen prior to notifying the physician. Irrigation of the NG tube should not be performed because the additional fluid may be spilled into the peritoneal cavity, and the client should be placed in a position of comfort, usually on the side with the head slightly elevated.

A physician has written an order for ranitidine (Zantac), 300 mg once daily. The nurse schedules the medication for which time? -At bedtime -After lunch -With supper -Before breakfast

-At bedtime Currently, the most commonly used therapy for peptic ulcers is a combination of antibiotics, proton-pump inhibitors, and bismuth salts that suppress or eradicate H. pylori. Recommended therapy for 10 to 14 days includes triple therapy with two antibiotics (eg, metronidazole [Flagyl] or amoxicillin [Amoxil] and clarithromycin [Biaxin]) plus a proton-pump inhibitor (eg, lansoprazole [Prevacid], omeprazole [Prilosec], or rabeprazole [Aciphex]), or quadruple therapy with two antibiotics (metronidazole and tetracycline) plus a proton-pump inhibitor and bismuth salts (Pepto-Bismol). Research is being conducted to develop a vaccine against H. pylori.

A client with peptic ulcer disease wants to know nonpharmacologic ways that he can prevent recurrence. Which of the following measures would the nurse recommend? Select all that apply. -Smoking cessation -Substitution of coffee with decaffeinated products -Avoidance of alcohol -Eating whenever hungry -Following a regular schedule for rest, relaxation, and meals

-Avoidance of alcohol -Smoking cessation -Following a regular schedule for rest, relaxation, and meals The likelihood of recurrence is reduced if the client avoids smoking, coffee (including decaffeinated coffee) and other caffeinated beverages, and alcohol. It is important to counsel the client to eat meals at regular times and in a relaxed setting and to avoid overeating.

The nurse is evaluating a client's ulcer symptoms to differentiate ulcer as duodenal or gastric. Which symptom should the nurse at attribute to a duodenal ulcer? -Vomiting -Hemorrhage -Awakening in pain -Weight loss

-Awakening in pain The client with a gastric ulcer often awakens between 1 to 2 AM with pain, and ingestion of food brings relief. Vomiting is uncommon in clients with duodenal ulcer. Hemorrhage is less likely in clients with duodenal ulcer than those with gastric ulcer. The client with a duodenal ulcer may experience weight gain.

A patient comes to the clinic with the complaint, "I think I have an ulcer." What is a characteristic associated with peptic ulcer pain that the nurse should inquire about? Select all that apply. -Burning sensation localized in the back or mid-epigastrium -Feeling of emptiness that precedes meals from 1 to 3 hours -Severe gnawing pain that increases in severity as the day progresses -Pain that radiates to the shoulder or jaw -Vomiting without associated nausea

-Burning sensation localized in the back or mid-epigastrium -Feeling of emptiness that precedes meals from 1 to 3 hours -Severe gnawing pain that increases in severity as the day progresses As a rule, the patient with an ulcer complains of dull, gnawing pain or a burning sensation in the mid-epigastrium or the back. Although vomiting is rare in uncomplicated peptic ulcer, it may be a symptom of a complication of an ulcer.

Which of the following clients is at highest risk for peptic ulcer disease? -Client with blood type A -Client with blood type B -Client with blood type AB -Client with blood type O

-Client with blood type O Clients with blood type O are more susceptible to peptic ulcers than those with blood types A, B, and AB.

A morbidly obese client asks the nurse if medications are available to assist with weight loss. The nurse knows that the client would not be a candidate for phentermine if the following is part of the client's health history: -Coronary artery disease -Diabetes -Use of lithium -Peptic ulcer disease

-Coronary artery disease Phentermine, which requires a prescription, stimulates central noradrenergic receptors, causing appetite suppression. It may increase blood pressure and should not be taken by people with a history of heart disease, uncontrolled hypertension, hyperthyroidism, or glaucoma.

Which ulcer is associated with extensive burn injury? -Cushing ulcer -Curling ulcer -Peptic ulcer -Duodenal ulcer

-Curling ulcer Curling ulcer is frequently observed about 72 hours after extensive burns and involves the antrum of the stomach or the duodenum.

A nurse caring for a patient in a burn treatment center knows to assess for the presence of which of the following types of ulcer about 72 hours post injury? -Peptic -Cushing's -Gastric -Curling's

-Curling's Curling's ulcer results from a complication from severe burns that causes reduced plasma volume that affects the gastric mucosa. Cushing's ulcer is produced by elevated intracranial pressure and is common with head injuries and brain trauma.

A client experienced extensive burns and 72 hours later has developed an ulcer. Which of the following types of ulcer is most likely in this client? -Stress ulcer -Curling's ulcer -Cushing's ulcer -Hashimoto's ulcer

-Curling's ulcer Curling's ulcer is frequently observed about 72 hours after extensive burns and involves the antrum of the stomach or duodenum.

A patient sustained second- and third-degree burns over 30% of the body surface area approximately 72 hours ago. What type of ulcer should the nurse be alert for while caring for this patient? -Curling's ulcer -Peptic ulcer -Esophageal ulcer -Meckel's ulcer

-Curling's ulcer Curling's ulcer is frequently observed about 72 hours after extensive burns and involves the antrum of the stomach or the duodenum.

A patient is complaining of diarrhea after having bariatric surgery. What nonpharmacologic treatment can the nurse suggest to decrease the incidence of diarrhea? -Decrease the fat content in the diet. -Increase the fiber content in the diet. -Decrease the amount of fluid the patient is drinking. -Increase the protein content in the diet.

-Decrease the fat content in the diet. Patients may complain of either diarrhea or constipation postprocedure. Diarrhea is more common an occurrence post bariatric surgery, particularly after malabsorptive procedures (Mechanick et al., 2008). Both may be prevented if the patient consumes a nutritious diet that is high in fiber. Steatorrhea also may occur as a result of rapid gastric emptying, which prevents adequate mixing with pancreatic and biliary secretions. In mild cases, reducing the intake of fat and administering an antimotility medication (e.g., loperamide [Imodium]) may control symptoms.

Symptoms associated with pyloric obstruction include all of the following except: -Anorexia -Diarrhea -Nausea and vomiting -Epigastric fullness

-Diarrhea Pyloric obstruction, also called gastric outlet obstruction (GOO), occurs when the area distal to the pyloric sphincter becomes scarred and stenosed from spasm or edema or from scar tissue that forms when an ulcer alternately heals and breaks down. The client may have nausea and vomiting, constipation, epigastric fullness, anorexia, and, later, weight loss.

Which of the following appears to be a significant factor in the development of gastric cancer? -Diet -Age -Ethnicity -Gender

-Diet Diet seems to be a significant factor: a diet high in smoked, salted, or pickled foods and low in fruits and vegetables may increase the risk of gastric cancer. The typical patient with gastric cancer is between 50 and 70 years of age. Men have a higher incidence than women. Native Americans, Hispanic Americans, and African Americans are twice as likely as Caucasian Americans to develop gastric cancer.

A patient who had a Roux-en-Y bypass procedure for morbid obesity ate a chocolate chip cookie after a meal. After ingestion of the cookie, the patient complained of cramping pains, dizziness, and palpitation. After having a bowel movement, the symptoms resolved. What should the patient be educated about regarding this event? -Gastric outlet obstruction -Dumping syndrome -Bile reflux -Celiac disease

-Dumping syndrome Dumping syndrome is an unpleasant set of vasomotor and GI symptoms that occur in up to 76% of patients who have had bariatric surgery. Early symptoms include a sensation of fullness, weakness, faintness, dizziness, palpitations, diaphoresis, cramping pains, and diarrhea. These symptoms resolve once the intestine has been evacuated (i.e., with defecation).

Clients with Type O blood are at higher risk for which of the following GI disorders? -Gastric cancer -Duodenal ulcers -Esophageal varices -Diverticulitis

-Duodenal ulcers Familial tendency also may be a significant predisposing factor. People with blood type O are more susceptible to peptic ulcers than are those with blood type A, B, or AB. Blood type is not a predisposing factor for gastric cancer, esophageal varices, and diverticulitis.

The nursing student approaches his instructor to discuss the plan of care for his client diagnosed with peptic ulcer disease. The student asks what is the most common site for peptic ulcer formation? The instructor would state which one of the following? -Duodenum -Esophagus -Pylorus -Stomach

-Duodenum Peptic ulcers occur mainly in the gastroduodenal mucosa because this tissue cannot withstand the digestive action of gastric acid (HCl) and pepsin.

A physician suspects that a client has peptic ulcer disease. With which of the following diagnostic procedures would the nurse most likely prepare to assist? -Barium study of the upper gastrointestinal tract -Endoscopy -Gastric secretion study -Stool antigen test

-Endoscopy Barium study of the upper GI tract may show an ulcer; however, endoscopy is the preferred diagnostic procedure because it allows direct visualization of inflammatory changes, ulcers, and lesions. Through endoscopy, a biopsy of the gastric mucosa and of any suspicious lesions can be obtained. Endoscopy may reveal lesions that, because of their size or location, are not evident on x-ray studies. Less invasive diagnostic measures for detecting H. pylori include serologic testing for antibodies against the H. pylori antigen, stool antigen test, and urea breath test.

Which medication is classified as a histamine-2 receptor antagonist? -Famotidine -Lansoprazole -Metronidazole -Esomeprazole

-Famotidine Famotidine is a histamine-2 receptor antagonist. Lansoprazole and esomeprazole are proton pump inhibitors (PPIs). Metronidazole is an antibiotic.

The nurse is assessing a client with an ulcer for signs and symptoms of hemorrhage. The nurse interprets which condition as a sign/symptom of possible hemorrhage? -Hematemesis -Bradycardia -Hypertension -Polyuria

-Hematemesis The nurse interprets hematemesis as a sign/symptom of possible hemorrhage from the ulcer. Other signs that can indicate hemorrhage include tachycardia, hypotension, and oliguria/anuria.

Which diagnostic test would be used first to evaluate a client with upper GI bleeding? -Upper GI series -Endoscopy -Hemoglobin and hematocrit -Arteriography

-Hemoglobin and hematocrit The nurse assesses for faintness or dizziness and nausea, which may precede or accompany bleeding. It is important to monitor vital signs frequently and to evaluate for tachycardia, hypotension, and tachypnea. Other nursing interventions include monitoring the hemoglobin and hematocrit, testing the stool for gross or occult blood, and recording hourly urinary output to detect anuria or oliguria (absence of or decreased urine production). If bleeding cannot be managed by the measures described, other treatment modalities such as endoscopy may be used to halt bleeding and avoid surgical intervention. There is debate regarding how soon endoscopy should be performed. Some clinicians believe endoscopy should be performed within the first 24 hours after hemorrhaging has ceased. Others believe endoscopy may be performed during acute bleeding, as long as the esophageal or gastric area can be visualized (blood may decrease visibility). An upper GI is less accurate than endoscopy and would not reveal a bleed. Arteriography is an invasive study associated with life-threatening complications and would not be used for an initial evaluation.

The nurse is cautiously assessing a client admitted with peptic ulcer disease because the most common complication that occurs in 10% to 20% of clients is: -Hemorrhage -Intractable ulcer -Perforation -Pyloric obstruction

-Hemorrhage Hemorrhage, the most common complication, occurs in 10% to 20% of clients with peptic ulcers. Bleeding may be manifested by hematemesis or melena. Perforation is erosion of the ulcer through the gastric serosa into the peritoneal cavity without warning. Intractable ulcer refers to one that is hard to treat, relieve, or cure. Pyloric obstruction, also called gastric outlet obstruction (GOO), occurs when the area distal to the pyloric sphincter becomes scarred and stenosed from spasm or edema or from scar tissue that forms when an ulcer alternately heals and breaks down.

A client with severe peptic ulcer disease has undergone surgery and is several hours postoperative. During assessment, the nurse notes that the client has developed cool skin, tachycardia, and labored breathing; the client also appears to be confused. Which of the following complications has the client most likely developed? -Hemorrhage -Penetration -Perforation -Pyloric obstruction

-Hemorrhage Signs of hemorrhage following surgery include cool skin, confusion, increased heart rate, labored breathing, and blood in the stool. Signs of penetration and perforation are severe abdominal pain, rigid and tender abdomen, vomiting, elevated temperature, and increased heart rate. Indicators of pyloric obstruction are nausea, vomiting, distended abdomen, and abdominal pain.

Review the following four examples of ideal body weight (IBW), actual weight, and body mass index (BMI). Using three criteria for each example, select the body weight that indicates morbid obesity. -IBW = 150 lbs; weight = 190 lbs; BMI = 26 kg/m2 -IBW = 132 lbs; weight = 184 lbs; BMI = 28 kg/m2 -IBW = 175 lbs; weight = 265 lbs; BMI = 29 kg/m2 -IBW = 145 lbs; weight = 290 lbs; BMI = 31 kg/m2

-IBW = 145 lbs; weight = 290 lbs; BMI = 31 kg/m2 The criteria for morbid obesity are a body weight that is twice IBW and a BMI that exceeds 30 kg/m2.

During assessment of a patient with gastritis, the nurse practitioner attempts to distinguish acute from chronic pathology. One criteria, characteristic of gastritis would be the: -Immediacy of the occurrence. -Presence of vomiting. -Frequency of abdominal discomfort. -Incidence of anorexia.

-Immediacy of the occurrence. Acute gastritis usually develops quickly, whereas chronic gastritis results from prolonged inflammation of the stomach.

The client has been taking famotidine (Pepcid) at home. The nurse prepares a teaching plan for the client indicating that the medication acts primarily to achieve which of the following? -Inhibit gastric acid secretions. -Neutralize acid in the stomach. -Shorten the time required for digestion in the stomach. -Improve the mixing of foods and gastric secretions.

-Inhibit gastric acid secretions. Famotidine is useful for treating and preventing ulcers and managing gastroesophageal reflux disease. It functions by inhibiting the action of histamine at the H-2 receptor site located in the gastric parietal cells, thus inhibiting gastric acid secretion.

A 66-year-old African-American client has recently visited a physician to confirm a diagnosis of gastric cancer. The client has a history of tobacco use and was diagnosed 10 years ago with pernicious anemia. He and his family are shocked about the possibility of cancer because he was asymptomatic prior to recent complaints of pain and multiple gastrointestinal symptoms. On the basis of knowledge of disease progression, the nurse assumes that organs adjacent to the stomach are also affected. Which of the following organs may be affected? Choose all that apply. -Liver -Pancreas -Bladder -Duodenum -Lungs

-Liver -Pancreas -Duodenum Most gastric cancers are adenocarcinomas; they can occur anywhere in the stomach. The tumor infiltrates the surrounding mucosa, penetrating the wall of the stomach and adjacent organs and structures. The liver, pancreas, esophagus, and duodenum are often already affected at the time of diagnosis. Metastasis through lymph to the peritoneal cavity occurs later in the disease.

Rebleeding may occur from a peptic ulcer and often warrants surgical interventions. Signs of bleeding include which of the following? -Mental confusion -Bradycardia -Bradypnea -Hypertension

-Mental confusion Signs of bleeding include tachycardia, tachypnea, hypotension, mental confusion, thirst, and oliguria.

Which is a true statement regarding gastric cancer? -Most clients are asymptomatic during the early stage of the disease. -Women have a higher incidence of gastric cancer. -The prognosis for gastric cancer is good. -Most cases are discovered before metastasis.

-Most clients are asymptomatic during the early stage of the disease. Most clients are asymptomatic during the early stage of the disease. Men have a higher incidence of gastric cancer. The prognosis is poor because the diagnosis is usually made late because most clients are asymptomatic during the early stage. Most cases of gastric cancer are discovered only after local invasion has advanced or metastases are present.

The nurse in the ED admits a client with suspected gastric outlet obstruction. The client's symptoms include nausea and vomiting. The nurse anticipates that the physician will issue which order? -Pelvic x-ray -Stool specimen -Nasogastric tube insertion -Oral contrast

-Nasogastric tube insertion The nurse anticipates an order for nasogastric tube insertion to decompress the stomach. Pelvic x-ray, oral contrast, and stool specimens are not indicated at this time.

A client undergoes total gastrectomy. Several hours after surgery, the nurse notes that the client's nasogastric (NG) tube has stopped draining. How should the nurse respond? -Notify the physician. -Reposition the tube. -Irrigate the tube. -Increase the suction level.

-Notify the physician. The nurse should notify the physician because an NG tube that fails to drain during the postoperative period may be clogged, which could increase pressure on the suture site because fluid isn't draining adequately. Repositioning or irrigating an NG tube in a client who has undergone gastric surgery can disrupt the anastomosis. Increasing the level of suction may cause trauma to GI mucosa or the suture line.

Peptic ulcer disease occurs more frequently in people with which blood type? -A -B -AB -O

-O People with blood type O are more susceptible to peptic ulcers than those with blood type A, B, or AB.

Which medication classification represents a proton (gastric acid) pump inhibitor? -Omeprazole -Sucralfate -Famotidine -Metronidazole

-Omeprazole Omeprazole decreases gastric acid by slowing the hydrogen-potassium adenosine triphosphatase pump on the surface of the parietal cells. Sucralfate is a cytoprotective drug. Famotidine is a histamine-2 receptor antagonist. Metronidazole is an antibiotic, specifically an amebicide.

A nurse practitioner prescribes drug therapy for a patient with peptic ulcer disease. Choose the drug that can be used for 4 weeks and has a 90% chance of healing the ulcer. -Ranitidine (Zantac) -Cimetidine (Tagamet) -Famotidine (Pepcid) -Omeprazole (Prilosec)

-Omeprazole (Prilosec) Omeprazole (Prilosec) is a proton pump inhibitor that, if used according to the health care provider's directions, will result in healing in 90% of patients. The other drugs are H2 receptor antagonists that need to be used for 6 weeks.

A patient is in the hospital for the treatment of peptic ulcer disease. The nurse finds the patient vomiting and complaining of a sudden severe pain in the abdomen. The nurse then assesses a board-like abdomen. What does the nurse suspect these symptoms indicate? -The treatment for the peptic ulcer is ineffective. -A reaction to the medication given for the ulcer -Gastric penetration -Perforation of the peptic ulcer

-Perforation of the peptic ulcer Signs and symptoms of perforation include the following: Sudden, severe upper abdominal pain (persisting and increasing in intensity), which may be referred to the shoulders, especially the right shoulder, because of irritation of the phrenic nerve in the diaphragm; vomiting; collapse (fainting); extremely tender and rigid (boardlike) abdomen; and hypotension and tachycardia, indicating shock.

Which of the following manifestations are associated with a deficiency of vitamin B12? Select all that apply. -Pernicious anemia -Macrocytic anemia -Thrombocytopenia -Loss of hair -Lethargy

-Pernicious anemia -Macrocytic anemia -Thrombocytopenia Decreased vitamin B12 can result in pernicious anemia, macrocytic anemia, and thrombocytopenia. Decreased iron can result in lethargy and loss of hair.

The nurse is teaching a client with peptic ulcer disease who has been prescribed misoprostol (Cytotec). What information from the nurse would be most accurate about misoprostol? -Works best when taken on an empty stomach -Increases the speed of gastric emptying -Prevents ulceration in clients taking nonsteroidal anti-inflammatory drugs (NSAIDs) -Decreases mucus production

-Prevents ulceration in clients taking nonsteroidal anti-inflammatory drugs (NSAIDs) Misoprostol is a synthetic prostaglandin that protects the gastric mucosa against ulceration and is used in clients who take NSAIDs. Misoprostol should be taken with food. It does not improve emptying of the stomach, and it increases (not decreases) mucus production.

The nurse advises the patient who has just been diagnosed with acute gastritis to: -Take an emetic to rid the stomach of the irritating products. -Refrain from food until the GI symptoms subside. -Restrict food and fluids for 12 hours. -Restrict all food for 72 hours to rest the stomach.

-Refrain from food until the GI symptoms subside. It usually takes 24 to 48 hours for the stomach to recover from an attack. Refraining from food until symptoms subside is recommended, but liquids should be taken in moderation. Emetics and vomiting can cause damage to the esophagus.

Which of the following is the most successful treatment for gastric cancer? -Removal of the tumor -Chemotherapy -Radiation -Palliation

-Removal of the tumor There is no successful treatment for gastric carcinoma except removal of the tumor. If the tumor can be removed while it is still localized to the stomach, the patient may be cured. If the tumor has spread beyond the area that can be excised, cure is less likely.

Which of the following medications used for obesity improves cardiovascular disease risk factors in obese patients with metabolic syndrome? -Rimonabant (Acomplia) -Orlistat (Xenical) -Alli -Sibutramine (Meridia)

-Rimonabant (Acomplia) Acomplia is the newest medication used to treat obesity. It stimulates weight reduction and improves cardiovascular disease risk factors in obese patients with metabolic syndrome. Meridia was recently pulled from the market because of the increased risk of heart attack and stroke associated with this medication. Orlistat, available by prescription and over the counter as Alli, reduces caloric intake by binding to gastric and pancreatic lipase to prevent digestion of fats.

A health care provider counsels a patient about bariatric surgery. He recommends the Roux-en-Y gastric bypass. The nurse explains to the patient that this procedure involves which of the following? -Gastroplasty with a vertical band allowing for a pouch with a 15 to 20 mL capacity -Biliopancreatic diversion with a duodenal switch -Separation of the jejunum with an anastomosis -Gastric banding that incorporates a prosthetic device to restrict oral intake

-Separation of the jejunum with an anastomosis The Roux-en-Y gastric bypass is recommended for long-term weight loss because it uses a combined restrictive and malabsorptive procedure. Refer to Figure 23-3 (A to D) in the text.

Which of the following are characteristics associated with the Zollinger-Ellison syndrome (ZES)? Select all that apply. -Constipation -Hypocalcemia -Severe peptic ulcers -Extreme gastric hyperacidity -Gastrin-secreting tumors of the pancreas

-Severe peptic ulcers -Extreme gastric hyperacidity -Gastrin-secreting tumors of the pancreas ZES consists of severe peptic ulcers, extreme gastric hyperacidity, and gastrin-secreting benign or malignant tumors of the pancreas. Diarrhea and steatorrhea may be evident. The client may have co-existing parathyroid adenomas or hyperplasia and may therefore exhibit signs of hypercalcemia.

A client is recovering from gastric surgery. Toward what goal should the nurse progress the client's enteral intake? -Three meals and 120 ml fluid daily -Three meals and three snacks and 120 mL fluid daily -Six small meals and 120 mL fluid daily -Six small meals daily with 120 mL fluid between meals

-Six small meals daily with 120 mL fluid between meals After the return of bowel sounds and removal of the nasogastric tube, the nurse may give fluids, followed by food in small portions. Foods are gradually added until the client can eat six small meals a day and drink 120 mL of fluid between meals.

Which is an accurate statement regarding gastric cancer? -The incidence of stomach cancer continues to decrease in the United States. -Most gastric cancer-related deaths occur in people younger than 40 years. -Females have a higher incidence of gastric cancers than males. -A diet high in smoked foods and low in fruits and vegetables may decrease the risk of gastric cancer.

-The incidence of stomach cancer continues to decrease in the United States. While the incidence in the United States continues to decrease, gastric cancer still accounts for 10,700 deaths annually. While gastric cancer deaths occasionally occur in younger people, most occur in people older than 40 years of age. Males have a higher incidence of gastric cancers than females. More accurately, a diet high in smoked foods and low in fruits and vegetables may increase the risk of gastric cancer.

A nurse is providing care for a client recovering from gastric bypass surgery. During assessment, the client exhibits pallor, perspiration, palpitations, headache, and feelings of warmth, dizziness, and drowsiness. The client reports eating 90 minutes ago. The nurse suspects: -Vasomotor symptoms associated with dumping syndrome -Dehiscence of the surgical wound -Peritonitis -A normal reaction to surgery

-Vasomotor symptoms associated with dumping syndrome Early manifestations of dumping syndrome occur 15 to 30 minutes after eating. Signs and symptoms include vertigo, tachycardia, syncope, sweating, pallor, palpitations, diarrhea, nausea, and the desire to lie down. Dehiscence of the surgical wound is characterized by pain and a pulling or popping feeling at the surgical site. Peritonitis presents with a rigid, boardlike abdomen, tenderness, and fever. The client's signs and symptoms aren't a normal reaction to surgery.

The nurse is caring for a client with chronic gastritis. The nurse monitors the client knowing that this client is at risk for which vitamin deficiency? -Vitamin A -Vitamin B12 -Vitamin C -Vitamin E

-Vitamin B12 Clients with chronic gastritis from vitamin deficiency usually have evidence of malabsorption of vitamin B12 caused by the production of antibodies that interfere with the binding of vitamin B12 to intrinsic factor. However, some clients with chronic gastritis have no symptoms. Vitamins A, C, and E are not affected by gastritis.

Which statement correctly identifies a difference between duodenal and gastric ulcers? -Malignancy is associated with duodenal ulcer. -Weight gain may occur with a gastric ulcer. -A gastric ulcer is caused by hypersecretion of stomach acid. -Vomiting is uncommon in clients with duodenal ulcers.

-Vomiting is uncommon in clients with duodenal ulcers. Vomiting is uncommon in clients diagnosed with duodenal ulcer. Malignancy is associated with a gastric ulcer. Weight gain may occur with a duodenal ulcer. Duodenal ulcers cause hypersecretion of stomach acid.

The nurse is conducting preoperative teaching to a patient prior to bariatric surgery. The nurse includes that the average weight loss of patients after bariatric surgery is which percent of their presurgical body weight?

25 to 35% The nurse includes that the average weight loss of a patients after bariatric surgery is 25 to 35% of their presurgical body weight.

After a client received a diagnosis of gastric cancer, the surgical team decides that a Billroth II would be the best approach to treatment. The nurse explains to the family that this procedure involves: -Limited resection in the distal portion of the stomach and removal of about 25% of the stomach -Wide resection of the middle and distal portions of the stomach with removal of about 75% of the stomach -Proximal subtotal gastrectomy -Total gastrectomy and esophagogastrectomy

-Wide resection of the middle and distal portions of the stomach with removal of about 75% of the stomach The Billroth I involves a limited resection and offers a lower cure rate than the Billroth II. The Billroth II procedure is a wider resection that involves removing approximately 75% of the stomach and decreases the possibility of lymph node spread or metastatic recurrence. A proximal subtotal gastrectomy may be performed for a resectable tumor located in the proximal portion of the stomach or cardia. A total gastrectomy or an esophagogastrectomy is usually performed in place of this procedure to achieve a more extensive resection.

A nurse is teaching a group of middle-aged men about peptic ulcers. When discussing risk factors for peptic ulcers, the nurse should mention: -a sedentary lifestyle and smoking. -a history of hemorrhoids and smoking. -alcohol abuse and a history of acute renal failure. -alcohol abuse and smoking.

-alcohol abuse and smoking. The nurse should mention that risk factors for peptic (gastric and duodenal) ulcers include alcohol abuse, smoking, and stress. A sedentary lifestyle and a history of hemorrhoids aren't risk factors for peptic ulcers. Chronic renal failure, not acute renal failure, is associated with duodenal ulcers.

A client is admitted to the health care facility with a diagnosis of a bleeding gastric ulcer. The nurse expects this client's stools to be: -coffee-ground-like. -clay-colored. -black and tarry. -bright red.

-black and tarry. Black, tarry stools are a sign of bleeding high in the GI tract, as from a gastric ulcer, and result from the action of digestive enzymes on the blood. Vomitus associated with upper GI tract bleeding commonly is described as coffee-ground-like. Clay-colored stools are associated with biliary obstruction. Bright red stools indicate lower GI tract bleeding.

The Zollinger-Ellison syndrome (ZES) consists of severe peptic ulcers, extreme gastric hyperacidity, and gastrin-secreting benign or malignant tumors of the pancreas. The nurse recognizes that an agent that is used to decrease bleeding and decrease gastric acid secretions is -ranitidine (Zantac) -omeprazole (Prilosec) -vasopressin (Pitressin) -octreotide (Sandostatin)

-octreotide (Sandostatin) For patients with ZES, hypersecretion of acid may be controlled with high doses of H2 receptor antagonists. These clients may require twice the normal dose, and dosages usually need to be increased with prolonged use. Octreotide (Sandostatin), a medication that suppresses gastrin levels, also may be prescribed.

A nurse is caring for a client who is undergoing a diagnostic workup for a suspected GI problem. The client reports gnawing epigastric pain following meals and heartburn. The nurse suspects the client has: -peptic ulcer disease. -ulcerative colitis. -appendicitis. -diverticulitis.

-peptic ulcer disease. Peptic ulcer disease is characterized by dull, gnawing pain in the midepigastrium or the back that worsens with eating. Ulcerative colitis is characterized by exacerbations and remissions of severe bloody diarrhea. Appendicitis is characterized by epigastric or umbilical pain along with nausea, vomiting, and low-grade fever. Pain caused by diverticulitis is in the left lower quadrant and has a moderate onset. It's accompanied by nausea, vomiting, fever, and chills.

A nurse who provides care in an ambulatory clinic integrates basic cancer screening into admission assessments. What client most likely faces the highest immediate risk of oral cancer?

A 65-year-old man with alcoholism who smokes

A patient is scheduled for a Billroth I procedure for ulcer management. What does the nurse understand will occur when this procedure is performed?

A partial gastrectomy is performed with anastomosis of the stomach segment to the duodenum.

A patient is scheduled for a Billroth I procedure for ulcer management. What does the nurse understand will occur when this procedure is performed?

A partial gastrectomy is performed with anastomosis of the stomach segment to the duodenum. A Billroth I procedure involves removal of the lower portion of the antrum of the stomach (which contains the cells that secrete gastrin) as well as a small portion of the duodenum and pylorus. The remaining segment is anastomosed to the duodenum.

A patient is scheduled for a Billroth I procedure for ulcer management. What does the nurse understand will occur when this procedure is performed? The antral portion of the stomach is removed and a vagotomy is performed. A partial gastrectomy is performed with anastomosis of the stomach segment to the duodenum. A sectioned portion of the stomach is joined to the jejunum. The vagus nerve is cut and gastric drainage is established.

A partial gastrectomy is performed with anastomosis of the stomach segment to the duodenum. Explanation: A Billroth I procedure involves removal of the lower portion of the antrum of the stomach (which contains the cells that secrete gastrin) as well as a small portion of the duodenum and pylorus. The remaining segment is anastomosed to the duodenum. A vagotomy severs the vagus nerve; a Billroth I procedure may be performed in conjunction with a vagotomy. If the remaining part of the stomach is anastomosed to the jejunum, the procedure is a Billroth II.

A client reports to the clinic, stating that she rapidly developed headache, abdominal pain, nausea, hiccuping, and fatigue about 2 hours ago. For dinner, she ate buffalo chicken wings and beer. Which of the following medical conditions is most consistent with the client's presenting problems?

Acute gastritis

The nurse is evaluating a client's ulcer symptoms to differentiate ulcer as duodenal or gastric. Which symptom should the nurse at attribute to a duodenal ulcer? Vomiting Constipation Hemorrhage Awakening in pain

Awakening in pain Explanation: The client with a duodenal ulcer is more likely to awaken with pain during the night than is the client with a gastric ulcer. Vomiting, constipation, diarrhea, and bleeding are symptoms common to both gastric and duodenal ulcers.

A client is scheduled for removal of the lower portion of the antrum of the stomach and a small portion of the duodenum and pylorus. What surgical procedure will the nurse prepare the client for?

Billroth I

Which term describes a reddened, circumscribed lesion that ulcerates and becomes crusted and is a primary lesion of syphilis?

Chancre

A patient has a BMI ranger greater than 40 kg/m2. What would this patient's obesity classification be?

Class III

Which term refers to the first portion of the small intestine?

Duodenum

A client who reports increasing difficulty swallowing, weight loss, and fatigue is diagnosed with esophageal cancer. Because this client has difficulty swallowing, what should the nurse assign highest priority to?

Maintaining a patent airway

A client has a gastrostomy tube that has been placed to drain stomach contents by low intermittent suction. What is the nurse's priority during this aspect of the client's care?

Measure and record drainage.

An obstruction of the lumen of the intesting due to intussusceptions, tumors, stenosis, strictures, adhesions, or hernias Diverticulitis Diverticulosis Mechanical bowel obstruction NoDiverticulitis Diverticulosis Mechanical bowel obstruction Nonmechanical (functional, paralytic, ileus) bowel obstructionnmechanical (functional, paralytic, ileus) bowel obstruction

Mechanical bowel obstruction

Which is a true statement regarding gastric cancer?

Most clients are asymptomatic during the early stage of the disease.

The nurse is caring for a patient after bariatric surgery who experiences symptoms of gastric outlet obstruction. Which of the following are contraindicated?

NG tube

Peptic ulcer disease occurs more frequently in people with which blood type?

O

A nurse practitioner prescribes drug therapy for a patient with peptic ulcer disease. Choose the drug that can be used for 4 weeks and has a 90% chance of healing the ulcer. Cimetidine Omeprazole Nizatidine Famotidine

Omeprazole Explanation: Omeprazole (Prilosec) is a proton pump inhibitor that, if used according to the health care provider's directions, will result in healing in 90% of patients. The other drugs are H2 receptor antagonists that need to be used for 6 weeks.

Which of the following are classified as a histamine-2 receptor antagonist?

Pepcid

A client is preparing for discharge to home following a partial gastrectomy and vagotomy. Which is the best rationale for the client being taught to lie down for 30 minutes after each meal?

Slows gastric emptying

A client is admitted to the hospital with an exacerbation of chronic gastritis. When assessing the client's nutritional status, the nurse should expect to find what type of deficiency?

vitamin b12

The nurse is preparing a teaching tool about delayed release proton pump inhibitors used to treat duodenal ulcer disease caused by H. pylori. Which statement will the nurse include that would apply to most types of proton pump inhibitor prescribed to treat this condition?

The medication is to be swallowed whole and taken before mealsInteract with the client in a relaxed manner. Help identify the client's current stressors. Discuss potential coping techniques with the client. Offer information about relaxation methods.

The nurse is inserting a nasogastric tube and the patient begins coughing and is unable to speak. What does the nurse suspect has occurred?

The nurse has inadvertently inserted the tube into the trachea.

Diagnostic testing of a client with a history of dyspepsia and abdominal pain has resulted in a diagnosis of gastric cancer. The nurse's anticipatory guidance should include what information?

The possibility of surgery, chemotherapy and radiotherapy

Why are antacids administered regularly, rather than as needed, in peptic ulcer disease?

To keep gastric pH at 3.0 to 3.5 To maintain a gastric pH of 3.0 to 3.5 throughout each 24-hour period, regular (not as needed) doses of an antacid are needed to treat peptic ulcer disease. Frequent administration of an antacid tends to decrease client compliance rather than promote it. Antacids don't regulate bowel patterns, and they decrease pepsin activity.

A nurse is providing care for a client recovering from gastric bypass surgery. During assessment, the client exhibits pallor, perspiration, palpitations, headache, and feelings of warmth, dizziness, and drowsiness. The client reports eating 90 minutes ago. The nurse suspects:

Vasomotor symptoms associated with dumping syndrome

The nurse is caring for a client with chronic gastritis. The nurse monitors the client knowing that this client is at risk for which vitamin deficiency?

Vitamin B12 Clients with chronic gastritis from vitamin deficiency usually have evidence of malabsorption of vitamin B12 caused by the production of antibodies that interfere with the binding of vitamin B12 to intrinsic factor. However, some clients with chronic gastritis have no symptoms. Vitamins A, C, and E are not affected by gastritis.

A client recovering from a total gastrectomy has a low red blood cell count. Which medication will the nurse expect to be prescribed for this client?

Vitamin B12 injections

Which statement correctly identifies a difference between duodenal and gastric ulcers?

Vomiting is uncommon in clients with duodenal ulcers.

A nurse is preparing to provide care for a client whose exacerbation of ulcerative colitis has required hospital admission. During an exacerbation of this health problem, the nurse would anticipate that the client's stools will have what characteristics? Watery with blood and mucus Hard and black formed stool Dry and streaked with blood Loose with visible fatty streaks

Watery with blood and mucus

After a client received a diagnosis of gastric cancer, the surgical team decides that a Billroth II would be the best approach to treatment. The nurse explains to the family that this procedure involves:

Wide resection of the middle and distal portions of the stomach with removal of about 75% of the stomach The Billroth I involves a limited resection and offers a lower cure rate than the Billroth II. The Billroth II procedure is a wider resection that involves removing approximately 75% of the stomach and decreases the possibility of lymph node spread or metastatic recurrence. A proximal subtotal gastrectomy may be performed for a resectable tumor located in the proximal portion of the stomach or cardia. A total gastrectomy or an esophagogastrectomy is usually performed in place of this procedure to achieve a more extensive resection.

A nurse is teaching a group of middle-aged men about peptic ulcers. When discussing risk factors for peptic ulcers, the nurse should mention:

alcohol abuse and smoking.

The nurse is creating a discharge plan of care for a client with a peptic ulcer. The nurse tells the client to avoid

decaffeinated coffee.

A client receives a local anesthetic to suppress the gag reflex for a diagnostic procedure of the upper GI tract. The nurse determines which nursing intervention is advised for this client? monitor for any breathing related disorder or discomforts do not give any food and fluids until the gag reflex returns monitor for cramping or abdominal distention measure fluid output for at least 24 hours after the procedure

do not give any food and fluids until the gag reflex returns

A nurse is caring for a client who underwent a subtotal gastrectomy. To manage dumping syndrome, the nurse should advise the client to:

drink liquids only between meals

A nurse is caring for a client who underwent a subtotal gastrectomy. To manage dumping syndrome, the nurse should advise the client to:

drink liquids only between meals.

A nurse is caring for a client who underwent a subtotal gastrectomy. To manage dumping syndrome, the nurse should advise the client to: drink liquids only with meals. drink liquids only between meals. restrict fluid intake to 1 qt (1,000 ml)/day. don't drink liquids 2 hours before meals.

drink liquids only between meals. Explanation: A client who experiences dumping syndrome after a subtotal gastrectomy should be advised to ingest liquids between meals rather than with meals. Taking fluids between meals allows for adequate hydration, reduces the amount of bulk ingested with meals, and aids in the prevention of rapid gastric emptying. There is no need to restrict the amount of fluids, just the time when the client drinks fluids. Drinking liquids with meals increases the risk of dumping syndrome by increasing the amount of bulk and stimulating rapid gastric emptying. Small amounts of water are allowable before meals.

A nurse is providing follow-up teaching at a clinic visit for a client recovering from gastric resection. The client reports sweating, diarrhea, nausea, palpitations, and the desire to lie down 15 to 30 minutes after meals. The nurse suspects the client has? a normal reaction to surgery peritonitis dehiscence of the surgical wound. dumping syndrome

dumping syndrome

A nurse is caring for a client who underwent a subtotal gastrectomy 24 hours ago. The client has a nasogastric (NG) tube. The nurse should:

irrigate the NG tube gently with normal saline solution if ordered The nurse can gently irrigate the tube if ordered, but must be careful not to reposition it. Repositioning can cause bleeding. The nurse should apply suction continuously — not every hour. The nurse shouldn't clamp the NG tube postoperatively because secretions and gas will accumulate, stressing the suture line.

A client has received a diagnosis of oral cancer. During client education, the client expresses dismay at not having recognized any early signs or symptoms of the disease. The nurse tells the client that in early stages of this disease, it is usual to have:

no symptoms.

The nurse recognizes that the client diagnosed with a duodenal ulcer will likely experience: vomiting hemorrhage pain 2 to 3 hours after a meal weight loss

pain 2 to 3 hours after a meal

A nurse is caring for a client who is undergoing a diagnostic workup for a suspected GI problem. The client reports gnawing epigastric pain following meals and heartburn. The nurse suspects the client has:

peptic ulcer disease. Peptic ulcer disease is characterized by dull, gnawing pain in the midepigastrium or the back that worsens with eating. Ulcerative colitis is characterized by exacerbations and remissions of severe bloody diarrhea. Appendicitis is characterized by epigastric or umbilical pain along with nausea, vomiting, and low-grade fever. Pain caused by diverticulitis is in the left lower quadrant and has a moderate onset. It's accompanied by nausea, vomiting, fever, and chills

A patient has been diagnosed with acute gastritis and asks the nurse what could have caused it. What is the best response by the nurse? (Select all that apply.)

"It can be caused by ingestion of strong acids." "You may have ingested some irritating foods." "Is it possible that you are overusing aspirin."

A client with a peptic ulcer is about to begin a therapeutic regimen that includes a bland diet, antacids, and famotidine. Before the client is discharged, the nurse should provide which instruction? "Increase your intake of fluids containing caffeine." "Avoid aspirin and products that contain aspirin." "Stop taking the drugs when your symptoms subside." "Eat three balanced meals every day."

"Avoid aspirin and products that contain aspirin."

To prevent gastroesophageal reflux in a client with hiatal hernia, the nurse should provide which discharge instruction?

"Avoid coffee and alcoholic beverages."

A client is prescribed tetracycline to treat peptic ulcer disease. Which instruction would the nurse give the client?

"Be sure to wear sunscreen while taking this medicine."

The health care provider prescribes a combination of three drugs to treat peptic ulcer disease. The nurse, preparing to review the drug actions and side effects with the patient, understands that the triple combination should be in which order? -Antibiotics, prostaglandin E1 analogs, and bismuth salts -Proton pump inhibitors, prostaglandin E1 analogs, and bismuth salts -Bismuth salts, antibiotics, and proton pump inhibitors -Prostaglandin E1 analogs, antibiotics, and proton pump inhibitors

-Bismuth salts, antibiotics, and proton pump inhibitors Refer to Table 23-1 in the text to review the recommended triple combination.

A client comes to the clinic after developing a headache, abdominal pain, nausea, hiccupping, and fatigue about 2 hours ago. The client tells the nurse that the last food was buffalo chicken wings and beer. Which medical condition does the nurse find to be most consistent with the client's presenting problems?

Acute gastritis

A client has a family history of stomach cancer. Which of the following factors would further increase the client's risk for developing gastric cancer? Select all that apply.

Age 55 years Previous infection with H. pylori

30. A patient has been prescribed orlistat (Xenical) for the treatment of obesity. When providing relevant Test Bank - Brunner & Suddarth's Textbook of Medical-Surgical Nursing 14e (Hinkle 2017) 887 health education for this patient, the nurse should ensure the patient is aware of what potential adverse effect of treatment? A) Bowel incontinence B) Flatus with oily discharge C) Abdominal pain D) Heat intolerance

Ans: B Feedback: Side effects of orlistat include increased frequency of bowel movements, gas with oily discharge, decreased food absorption, decreased bile flow, and decreased absorption of some vitamins. This drug does not cause bowel incontinence, abdominal pain, or heat intolerance.

A patient has been diagnosed with peptic ulcer disease and the nurse is reviewing his prescribed medication regimen with him. What is currently the most commonly used drug regimen for peptic ulcers?

Antibiotics, proton pump inhibitors, and bismuth salts

A client's NG tube has become clogged after the nurse instilled a medication that was insufficiently crushed. The nurse has attempted to aspirate with a large-bore syringe, with no success. What should the nurse do next?

Attach a syringe filled with warm water and attempt an in-and-out motion of instilling and aspirating.

The nurse is evaluating a client's ulcer symptoms to differentiate ulcer as duodenal or gastric. Which symptom should the nurse at attribute to a duodenal ulcer?

Awakening in pain

The nurse is conducting an admission assessment and determines that the patient's BMI is 37. The nurse documents the BMI as being which of the following classes of obesity?

Class II

A client sustained second- and third-degree burns over 30% of the body surface area approximately 72 hours ago. What type of ulcer should the nurse be alert for while caring for this client? - Curling's ulcer - Peptic ulcer - Esophageal ulcer - Meckel's ulcer

Correct response: Curling's ulcer Explanation: Curling's ulcer is frequently observed about 72 hours after extensive burns and involves the antrum of the stomach or the duodenum. Peptic, esophageal, and Meckel's ulcers are not related to burn injuries.

A client with gastric cancer is having a resection. What is the nursing management priority for this client?

Correcting nutritional deficits Clients with gastric cancer commonly have nutritional deficits and may have cachexia. Therefore, correcting nutritional deficits is a top priority. Discharge planning before surgery is important, but correcting the nutritional deficits is a higher priority. Radiation therapy hasn't been proven effective for gastric cancer, and teaching about it preoperatively wouldn't be appropriate. Preventing DVT isn't a high priority before surgery, but it assumes greater importance after surgery.

Which ulcer is associated with extensive burn injury?

Curling ulcer

Which ulcer is associated with extensive burn injury?

Curling ulcer Curling ulcer is frequently observed about 72 hours after extensive burns and involves the antrum of the stomach or the duodenum.

Symptoms associated with pyloric obstruction include all of the following except:

Diarrhea Pyloric obstruction, also called gastric outlet obstruction (GOO), occurs when the area distal to the pyloric sphincter becomes scarred and stenosed from spasm or edema or from scar tissue that forms when an ulcer alternately heals and breaks down. The client may have nausea and vomiting, constipation, epigastric fullness, anorexia, and, later, weight loss.

A nurse is providing follow-up teaching at a clinic visit for a client recovering from gastric resection. The client reports sweating, diarrhea, nausea, palpitations, and the desire to lie down 15 to 30 minutes after meals. Based on the client's assessment, what will the nurse suspect?

Dumping syndrome

A patient who had a Roux-en-Y bypass procedure for morbid obesity ate a chocolate chip cookie after a meal. After ingestion of the cookie, the patient complained of cramping pains, dizziness, and palpitation. After having a bowel movement, the symptoms resolved. What should the patient be educated about regarding this event?

Dumping syndrome

A physician suspects that a client has peptic ulcer disease. With which of the following diagnostic procedures would the nurse most likely prepare to assist?

Endoscopy Endoscopy is the preferred diagnostic procedure because it allows direct visualization of inflammatory changes, ulcers, and lesions. Through endoscopy, a biopsy of the gastric mucosa and of any suspicious lesions can be obtained. Endoscopy may reveal lesions that, because of their size or location, are not evident on x-ray studies. Less invasive diagnostic measures for detecting H. pylori include serologic testing for antibodies against the H. pylori antigen, stool antigen test, and urea breath test.

The nurse is developing a plan of care for a patient with peptic ulcer disease. What nursing interventions should be included in the care plan? Select all that apply.

Frequently monitoring hemoglobin and hematocrit levels Observing stools and vomitus for color, consistency, and volume Checking the blood pressure and pulse rate every 15 to 20 minutes

The nurse is developing a plan of care for a patient with peptic ulcer disease. What nursing interventions should be included in the care plan? (Select all that apply.)

Frequently monitoring hemoglobin and hematocrit levels Observing stools and vomitus for color, consistency, and volume Checking the blood pressure and pulse rate every 15 to 20 minutes The nurse assesses the patient for faintness or dizziness and nausea, which may precede or accompany bleeding. The nurse must monitor vital signs frequently and evaluate the patient for tachycardia, hypotension, and tachypnea. Other nursing interventions include monitoring the hemoglobin and hematocrit, testing the stool for gross or occult blood, and recording hourly urinary output to detect anuria or oliguria (absence of or decreased urine production).

Which of the following surgical procedures for obesity utilizes a prosthetic device to restrict oral intake?

Gastric banding In gastric banding, a prosthetic device is used to restrict oral intake by creating a small pouch of 10 to 15 milliliters that empties through the narrow outlet into the remainder of the stomach. Roux-en-Y gastric bypass uses a horizontal row of staples across the fundus of the stomach to create a pouch with a capacity of 20 to 30 mL. Vertical-banded gastroplasty involves placement of a vertical row of staples along the lesser curvature of the stomach, creating a new, small gastric pouch. Biliopancreatic diversion with duodenal switch combines gastric restriction with intestinal malabsorption.

Which diagnostic test would be used first to evaluate a client with upper GI bleeding?

Hemoglobin and hematocrit

The nurse is cautiously assessing a client admitted with peptic ulcer disease because the most common complication that occurs in 10% to 20% of clients is:

Hemorrhage

Which of the following is the most common complication associated with peptic ulcer?

Hemorrhage Hemorrhage, the most common complication, occurs in 28% to 59% of patients with peptic ulcers. Vomiting, elevated temperature, and abdominal pain are not the most common complications of a peptic ulcer.

Review the following four examples of ideal body weight (IBW), actual weight, and body mass index (BMI). Using three criteria for each example, select the body weight that indicates morbid obesity.

IBW = 145 lbs; weight = 290 lbs; BMI = 31 kg/m2

The nurse visits the home of a client recovering from acute gastritis. Which observation indicates that teaching about the disorder was effective? Cup of caffeinated coffee on the kitchen table Medications placed in a pillbox Tomato sauce simmering on the stove Extinguished cigarettes in an ashtray

Medications placed in a pillbox Explanation: The client with acute gastritis should be instructed on methods of keeping track of medications such as placing the doses into a pillbox. The client should also be instructed about foods and substances that may cause gastritis, including nicotine, spicy seasoned foods, and caffeine. Cigarettes in the ashtray, tomato sauce, and caffeinated coffee indicate that additional teaching is required.

The nurse is aware that hemorrhage is a common complication of peptic ulcer disease. Therefore, assessment for indicators of bleeding is an important nursing responsibility. Which of the following are indicators of bleeding? Select all that apply.

Melena Tachypnea Thirst Mental confusion

Rebleeding may occur from a peptic ulcer and often warrants surgical interventions. Signs of bleeding include which of the following?

Mental confusion

A client is prescribed a histamine (H2)-receptor antagonist. The nurse understands that this might include which medication(s)? Select all that apply. Esomeprazole Famotidine Cimetidine Nizatidine Lansoprazole

Nizatidine Famotidine Cimetidine Explanation: H2-receptor antagonists suppress secretion of gastric acid, alleviate symptoms of heartburn, and assist in preventing complications of peptic ulcer disease. These medications also suppress gastric acid secretions and are used in active ulcer disease, erosive esophagitis, and pathological hypersecretory conditions. The other medications listed are proton-pump inhibitors.

A client is prescribed a histamine (H2)-receptor antagonist. The nurse understands that the following are H2-receptor antagonists. Choose all that apply.

Nizatidithene (Axid) Ranitidine (Zantac) Famotidine (Pepcid) Cimetidine (Tagamet)

Absence of peristalisis due to diabetes, neurologic disorders or manipulation of the bowel during surgery Diverticulitis Diverticulosis Mechanical bowel obstruction Nonmechanical (functional, paralytic, ileus) bowel obstruction

Nonmechanical (functional, paralytic, ileus) bowel obstruction

A nurse practitioner prescribes drug therapy for a patient with peptic ulcer disease. Choose the drug that can be used for 4 weeks and has a 90% chance of healing the ulcer. Ranitidine (Zantac) Cimetidine (Tagamet) Famotidine (Pepcid) Omeprazole (Prilosec)

Omeprazole (Prilosec) Omeprazole (Prilosec) is a proton pump inhibitor that, if used according to the health care provider's directions, will result in healing in 90% of patients. The other drugs are H2 receptor antagonists that need to be used for 6 weeks.

Which of the following represents the medication classification of a proton (gastric acid) pump inhibitor?

Omeprazole (Prilosec) Omeprazole decreases gastric acid by slowing the hydrogen-potassium adenosine triphosphatase pump on the surface of the parietal cells. Sucralfate is a cytoprotective drug. Famotidine is a histamine-2 receptor antagonist. Metronidazole is an antibiotic, specifically an amebicide.

Which of the following medications used for the treatment of obesity prevents the absorption of triglycerides?

Orlistat (Xenical) Orlistat (Xenical) prevents the absorption of triglycerides. Side effects of Xenical may include increased bowel movements, gas with oily discharge, decreased food absorption, decreased bile flow, and decreased absorption of some vitamins. Bupropion hydrochloride (Wellbutrin) is an antidepressant medication. Sibutramine hydrochloride (Meridia) inhibits the reuptake of serotonin and norepinephrine. Meridia decreases appetite. Fluoxetine hydrochloride (Prozac) has not been approved by the FDA for use in the treatment of obesity.

An elderly client comes into the emergency department reporting an earache. The client and has an oral temperature of 37.9° (100.2ºF) and otoscopic assessment of the ear reveals a pearly gray tympanic membrane with no evidence of discharge or inflammation. Which action should the triage nurse take next?

Palpate the client's parotid glands to detect swelling and tenderness.

The nurse is conducting a community education class on gastritis. The nurse includes that chronic gastritis caused by Helicobacter pylori is implicated in which disease/condition?

Peptic ulcers

A patient is in the hospital for the treatment of peptic ulcer disease. The nurse finds the patient vomiting and complaining of a sudden severe pain in the abdomen. The nurse then assesses a board-like abdomen. What does the nurse suspect these symptoms indicate?

Perforation of the peptic ulcer

A patient is in the hospital for the treatment of peptic ulcer disease. The nurse finds the patient vomiting and complaining of a sudden severe pain in the abdomen. The nurse then assesses a board-like abdomen. What does the nurse suspect these symptoms indicate?

Perforation of the peptic ulcer Signs and symptoms of perforation include the following: Sudden, severe upper abdominal pain (persisting and increasing in intensity), which may be referred to the shoulders, especially the right shoulder, because of irritation of the phrenic nerve in the diaphragm; vomiting; collapse (fainting); extremely tender and rigid (boardlike) abdomen; and hypotension and tachycardia, indicating shock.

Which of the following manifestations are associated with a deficiency of vitamin B12? Select all that apply.

Pernicious anemia Macrocytic anemia Thrombocytopenia

The nurse advises the patient who has just been diagnosed with acute gastritis to: Take an emetic to rid the stomach of the irritating products. Refrain from food until the GI symptoms subside. Restrict food and fluids for 12 hours. Restrict all food for 72 hours to rest the stomach.

Refrain from food until the GI symptoms subside. Explanation: It usually takes 24 to 48 hours for the stomach to recover from an attack. Refraining from food until symptoms subside is recommended, but liquids should be taken in moderation. Emetics and vomiting can cause damage to the esophagus.

Which of the following is the most successful treatment for gastric cancer? Chemotherapy Removal of the tumor Palliation Radiation

Removal of the tumor Explanation: There is no successful treatment for gastric carcinoma except removal of the tumor. If the tumor can be removed while it is still localized to the stomach, the patient may be cured. If the tumor has spread beyond the area that can be excised, cure is less likely.

The nurse is caring for a client with a duodenal ulcer and is relating the client's symptoms to the physiologic functions of the small intestine. What do these functions include? **Select all that apply Secretion of HCl (hydrochloric acid) Reabsorption of water Secretion of mucus Absorption of nutrients Movement of nutrients into the bloodstream

Secretion of mucus Absorption of nutrients Movement of nutrients into the bloodstream

A health care provider counsels a patient about bariatric surgery. He recommends the Roux-en-Y gastric bypass. The nurse explains to the patient that this procedure involves which of the following?

Separation of the jejunum with an anastomosis

A client will be undergoing abdominal computed tomography (CT) with contrast. The nurse has administered IV sodium bicarbonate and oral acetylcysteine before the study, as prescribed. What would indicate that these medications have had the desired therapeutic effect? The client's BUN and creatinine levels are within reference range following the CT The CT yields high-quality images The client's electrolytes are stable in the 48 hours following the CT The client's intake and output are in balance on the day after the CT

The client's BUN and creatinine levels are within reference range following the CT

After a client received a diagnosis of gastric cancer, the surgical team decides that a Billroth II would be the best approach to treatment. The nurse explains to the family that this procedure involves:

Wide resection of the middle and distal prtions of the stomach with removal of about 75% of the stomach. The Billroth I involves a limited resection and offers a lower cure rate than the Billroth II. The Billroth II procedure is a wider resection that involves removing approximately 75% of the stomach and decreases the possibility of lymph node spread or metastatic recurrence. A proximal subtotal gastrectomy may be performed for a resectable tumor located in the proximal portion of the stomach or cardia. A total gastrectomy or an esophagogastrectomy is usually performed in place of this procedure to achieve a more extensive resection.

A nurse is caring for a client who underwent a subtotal gastrectomy. To manage dumping syndrome, the nurse should advise the client to: restrict fluid intake to 1 qt (1,000 ml)/day. drink liquids only with meals. don't drink liquids 2 hours before meals. drink liquids only between meals.

drink liquids only between meals.

A nurse is caring for a client who underwent a subtotal gastrectomy. To manage dumping syndrome, the nurse should advise the client to:

drink liquids only between meals. A client who experiences dumping syndrome after a subtotal gastrectomy should be advised to ingest liquids between meals rather than with meals. Taking fluids between meals allows for adequate hydration, reduces the amount of bulk ingested with meals, and aids in the prevention of rapid gastric emptying. There is no need to restrict the amount of fluids, just the time when the client drinks fluids. Drinking liquids with meals increases the risk of dumping syndrome by increasing the amount of bulk and stimulating rapid gastric emptying. Small amounts of water are allowable before meals.

A nurse is providing follow-up teaching at a clinic visit for a client recovering from gastric resection. The client reports sweating, diarrhea, nausea, palpitations, and the desire to lie down 15 to 30 minutes after meals. The nurse suspects the client has

dumping syndrome. Early manifestations of dumping syndrome occur 15 to 30 minutes after eating. Signs and symptoms include vertigo, tachycardia, syncope, sweating, pallor, palpitations, diarrhea, nausea, and the desire to lie down. Dehiscence of the surgical wound is characterized by pain and a pulling or popping feeling at the surgical site. Peritonitis presents with a rigid, boardlike abdomen, tenderness, and fever. The client's signs and symptoms aren't a normal reaction to surgery.

The most common symptom of esophageal disease is

dysphagia.

An adult client is scheduled for an upper GI series that will use a barium swallow. What teaching should the nurse include when the client has completed the test? stool will be yellow for the first 24 hours postprocedure the barium may cause diarrhea for the next 24 hours fluids must be increased to facilitate the evacuation of the stool slight anal bleeding may be noted as the barium is passed

fluids must be increased to facilitate the evacuation of the stool

A nurse is caring for a client who is scheduled for a colonoscopy and whose preparation will include polyethylene glycol electrolyte lavage prior to the procedure. The presence of what health problem would contraindicate the use of this form of bowel preparation? colon cancer diverticulitis intestinal polyps inflammatory bowel disease

inflammatory bowel disease

The nurse is preparing to perform a client's abdominal assessment. What examination sequence should the nurse follow? inspection, auscultation, percussion, palpation inspection, palpation, auscultation, percussion inspection, percussion, palpation, auscultation inspection, palpation, percussion, auscultation

inspection, auscultation, percussion, palpation

The nurse recognizes that the client diagnosed with a duodenal ulcer will likely experience

pain 2 to 3 hours after a meal.

The most significant complication related to continuous tube feedings is

the increased potential for aspiration.

A client with a recent history of intermittent bleeding is undergoing capsule endoscopy to determine the source of the bleeding. When explaining this diagnostic test to the client, what advantage should the nurse describe? the test allows visualization of the entire peritoneal cavity the test allows for painless biopsy collection the test does not require fasting the test is noninvasive

the test is noninvasive

A nurse is caring for a newly admitted client with a suspected GI bleed. The nurse assesses the client's stool after a bowel movement and notes it to be a tarry-black color. This finding is suggestive of bleeding from what location? sigmoid colon upper GI tract large intestine anus or rectum

upper GI tract

A client is admitted to the hospital with an exacerbation of his chronic gastritis. When assessing his nutritional status, the nurse should expect a deficiency in:

vitamin B12 The nurse should expect vitamin B12 deficiency. Injury to the gastric mucosa causes gastric atrophy and impaired function of the parietal cells. These changes result in reduced production of intrinsic factor, which is necessary for the absorption of vitamin B12. Eventually, pernicious anemia will occur. Deficiencies in vitamins A, B6, and C aren't expected in a client with chronic gastritis.

A client is prescribed tetracycline to treat peptic ulcer disease. Which of the following instructions would the nurse give the client?

"Be sure to wear sunscreen while taking this medicine."

A client is prescribed tetracycline to treat peptic ulcer disease. Which instruction would the nurse give the client? "Be sure to wear sunscreen while taking this medicine." "Take the medication with milk." "You will not experience GI upset while taking this medication." "Do not drive when taking this medication."

"Be sure to wear sunscreen while taking this medicine." Explanation: Tetracycline may cause a photosensitivity reaction in clients. The nurse should caution the client to use sunscreen when taking this drug. Dairy products can reduce the effectiveness of tetracycline, so the nurse should not advise him or her to take the medication with milk. GI upset is possible with tetracycline administration. Administration of tetracycline does not necessitate driving restrictions.

A nurse is teaching a client with gastritis about the need to avoid the intake of caffeinated beverages. The client asks why this is so important. Which explanation from the nurse would be most accurate? "Caffeine can interfere with absorption of vitamin B12, which leads to anemia and further digestive problems." "Caffeine stimulates the central nervous system and thus gastric activity and secretions, which need to be minimized to promote recovery." "Caffeine intake can cause tears in your esophagus and intestines, which can lead to hemorrhage." "Caffeine increases the fluid volume in your system, which irritates your digestive organs."

"Caffeine stimulates the central nervous system and thus gastric activity and secretions, which need to be minimized to promote recovery." Explanation: Caffeine is a central nervous system stimulant that increases gastric activity and pepsin secretion. Caffeine is a diuretic that causes decreased fluid volume and potential dehydration. It does not lead to hemorrhage and does not interfere with absorption of vitamin B12.

A client admitted for treatment of a gastric ulcer is being prepared for discharge on antacid therapy. Discharge teaching should include which instruction?

"Continue to take antacids even if your symptoms subside." Antacids decrease gastric acidity and should be continued even if the client's symptoms subside. Because other medications may interfere with antacid action, the client should avoid taking antacids concomitantly with other drugs. If cardiac problems arise, the client should avoid antacids containing sodium, not magnesium. For optimal results, the client should take an antacid 1 hour before or 2 hours after meals

The nurse determines that teaching for the client with peptic ulcer disease has been effective when the client makes which statement?

"I have learned some relaxation strategies that decrease my stress."

The nurse determines that teaching for the client with peptic ulcer disease has been effective when the client states:

"I have learned some relaxation strategies that decrease my stress." The nurse assists the client to identify stressful or exhausting situations. A hectic lifestyle and an irregular schedule may aggravate symptoms and interfere with regular meals taken in relaxed settings along with the regular administration of medications. The client may benefit from regular rest periods during the day, at least during the acute phase of the disease. Biofeedback, hypnosis, behavior modification, massage, or acupuncture may be helpful.

After teaching a client who has had a Roux-en-Y gastric bypass, which client statement indicates the need for additional teaching?

"I need to drink 8 ounces of water before eating." After a Roux-en-Y gastric bypass, the client should not drink fluids with meals, withholding fluids for 15 minutes before eating to 90 minutes after eating. Chewing foods slowly and thoroughly, keeping total serving sizes to less than 1 cup, and choosing foods such as breads, cereals, and grains that provide less than 2 g of fiber per serving.

The nurse provides client education to a client about to undergo Urea breath testing. The nurse evaluates that the client understands the test when the client makes which statement? "First, I will drink a cherry flavored liquid." "The test will detect the presence of staph." "I should avoid antibiotics for 1 month before the test." "The test will detect the presence of oral cancer."

"I should avoid antibiotics for 1 month before the test."

A nurse is performing discharge teaching with a client who had a total gastrectomy. Which statement indicates the need for further teaching? "I'm going to visit my pastor weekly for a while." "I will have to take vitamin B12 shots up to 1 year after surgery." "I will weight myself each day and record the weight." "I will call my physician if I begin to have abdominal pain."

"I will have to take vitamin B12 shots up to 1 year after surgery." Explanation: After a total gastrectomy, a client will need to take vitamin B12 shots for life. Dietary B12 is absorbed in the stomach, and the inability to absorb it could lead to pernicious anemia. Visiting clergy for emotional support is normal after receiving a cancer diagnosis. This action should be encouraged by the nurse. It's appropriate for the client to call the physician if he experiences signs and symptoms of intestinal blockage or obstruction, such as abdominal pain. Because a client with a total gastrectomy will receive enteral feedings or parenteral feedings, he should weigh himself each day and keep a record of the weights.

The nurse instructs a client with acute gastritis on lifestyle modifications. Which client statement indicates that additional teaching is required? "I will reduce the amount of smoking." "I will switch to decaffeinated coffee." "I will increase my intake of fresh fruit." "I will avoid alcoholic beverages."

"I will reduce the amount of smoking." Explanation: Lifestyle modifications for the client with acute gastritis focus on smoking, alcohol intake, and caffeine. Smoking should be discouraged because the level of nicotine in gastric acid can be 10 times greater than in arterial blood and 80 times greater than venous blood. Alcohol intake should also be discouraged. Caffeinated beverages should also be discouraged because caffeine is a central nervous system stimulant that increases gastric activity and pepsin secretion. Increasing the intake of fresh fruit is not identified as a lifestyle modification for acute gastritis.

A client with an H. pylori infection asks why bismuth subsalicylate is prescribed. Which response will the nurse make? "It improves digestion in the stomach." "It aids in the healing of the stomach lining." "It enhances the function of the pyloric sphincter." "It helps propel food from the stomach into the duodenum."

"It aids in the healing of the stomach lining." Explanation: Bismuth subsalicylate suppresses H. pylori bacteria in the gastric mucosa and assists with healing of mucosal ulcers. It does not affect digestion, enhance the function of the pyloric sphincter, or propel food from the stomach into the duodenum.

A patient has been diagnosed with acute gastritis and asks the nurse what could have caused it. What is the best response by the nurse? (Select all that apply.)

"It can be caused by ingestion of strong acids." "You may have ingested some irritating foods." "Is it possible that you are overusing aspirin."

A client with gastric ulcers caused by H. pylori is prescribed metronidazole. Which client statement indicates to the nurse that teaching about this medication was effective?

"It might cause a metallic taste in my mouth."

A client with gastric ulcers caused by H. pylori is prescribed metronidazole. Which client statement indicates to the nurse that teaching about this medication was effective? "It might cause a metallic taste in my mouth." "My appetite may increase while taking this medication." "I can have an alcoholic drink in the evenings." "I can take this medication with my blood thinner."

"It might cause a metallic taste in my mouth." Explanation: Metronidazole is a synthetic antibacterial and antiprotozoal agent that assists with eradicating H. pylori bacteria in the gastric mucosa when given with other antibiotics and proton pump inhibitors. This medication may cause a metallic taste in the mouth. It should not be taken with anticoagulants as it will increase the blood thinning effects of warfarin. Alcohol should be avoided while taking this medication. This medication may cause anorexia and not an increased appetite.

A nurse is teaching a client who has experienced an episode of acute gastritis. The nurse knows further education is necessary when the client makes which statement? "I should limit alcohol intake, at least until symptoms subside." "I should feel better in about 24 to 36 hours." "My appetite should come back tomorrow." "Once I can eat again, I should stick with bland foods."

"My appetite should come back tomorrow." Explanation: The gastric mucosa is capable of repairing itself after an episode of gastritis. As a rule, the client recovers in about 1 day, although the appetite may be diminished for an additional 2 or 3 days. Acute gastritis is also managed by instructing the client to refrain from alcohol and food until symptoms subside. When the client can take nourishment by mouth, a nonirritating diet is recommended.

A client who suffered a stroke had an NG tube inserted to facilitate feeding shortly after admission. The client has since become comatose and the client's family asks the nurse why the health care provider is recommending the removal of the client's NG tube and the insertion of a gastrostomy tube. What is the nurse's best response?

"Regurgitation and aspiration are less likely."

A client with a peptic ulcer is diagnosed with Helicobacter pylori infection. The nurse is teaching the client about the medications prescribed, including metronidazole (Flagyl), omeprazole (Prilosec), and clarithromycin (Biaxin). Which statement by the client indicates the best understanding of the medication regimen?

"The medications will kill the bacteria and stop the acid production."

A client with a peptic ulcer is diagnosed with Heliobacter pylori infection. The nurse is teaching the client about the medications prescribed, including metronidazole (Flagyl), omeprazole (Prilosec), and clarithromycin (Biaxin). Which statement by the client indicates the best understanding of the medication regimen?

"The medications will kill the bacteria and stop the acid production." Currently, the most commonly used therapy for peptic ulcers is a combination of antibiotics, proton-pump inhibitors, and bismuth salts that suppress or eradicate H. pylori. Recommended therapy for 10 to 14 days includes triple therapy with two antibiotics (eg, metronidazole [Flagyl] or amoxicillin [Amoxil] and clarithromycin [Biaxin]) plus a proton-pump inhibitor (eg, lansoprazole [Prevacid], omeprazole [Prilosec], or rabeprazole [Aciphex]), or quadruple therapy with two antibiotics (metronidazole and tetracycline) plus a proton-pump inhibitor and bismuth salts (Pepto-Bismol). Research is being conducted to develop a vaccine against H. pylori.

A patient is scheduled for a Billroth I procedure for ulcer management. What does the nurse understand will occur when this procedure is performed? -A partial gastrectomy is performed with anastomosis of the stomach segment to the duodenum. -A sectioned portion of the stomach is joined to the jejunum. -The antral portion of the stomach is removed and a vagotomy is performed. -The vagus nerve is cut and gastric drainage is established.

-A partial gastrectomy is performed with anastomosis of the stomach segment to the duodenum. A Billroth I procedure involves removal of the lower portion of the antrum of the stomach (which contains the cells that secrete gastrin) as well as a small portion of the duodenum and pylorus. The remaining segment is anastomosed to the duodenum.

Which is a true statement regarding the nursing considerations in administration of metronidazole? -It may cause weight gain. -The drug should be given before meals. -Metronidazole decreases the effect of warfarin. -It leaves a metallic taste in the mouth.

-It leaves a metallic taste in the mouth. Metronidazole leaves a metallic taste in the mouth. It may cause anorexia and should be given with meals to decrease gastrointestinal upset. Metronidazole increases the blood-thinning effects of warfarin.

A nurse is monitoring a client with peptic ulcer disease. Which of the following assessment findings would most likely indicate perforation of the ulcer? Choose all that apply. -Tachycardia -Hypotension -Mild epigastric pain -A rigid, board-like abdomen -Diarrhea

-Tachycardia -Hypotension -A rigid, board-like abdomen Signs and symptoms of perforation include sudden, severe upper abdominal pain (persisting and increasing in intensity); pain, which may be referred to the shoulders, especially the right shoulder, because of irritation of the phrenic nerve in the diaphragm; vomiting; collapse (fainting); extremely tender and rigid (board-like) abdomen; and hypotension and tachycardia, indicating shock. Perforation is a surgical emergency.

The nurse is creating a discharge plan of care for a client with a peptic ulcer. The nurse tells the client to avoid -acetaminophen. -decaffeinated coffee. -skim milk. -octreotide.

-decaffeinated coffee. The nurse should include avoidance of decaffeinated coffee in the client's discharge teaching plan. Decaffeinated coffee is avoided to keep from overstimulating acid secretion.

A nurse is providing follow-up teaching at a clinic visit for a client recovering from gastric resection. The client reports sweating, diarrhea, nausea, palpitations, and the desire to lie down 15 to 30 minutes after meals. The nurse suspects the client has: -dumping syndrome. -dehiscence of the surgical wound. -peritonitis. -a normal reaction to surgery.

-dumping syndrome. Early manifestations of dumping syndrome occur 15 to 30 minutes after eating. Signs and symptoms include vertigo, tachycardia, syncope, sweating, pallor, palpitations, diarrhea, nausea, and the desire to lie down. Dehiscence of the surgical wound is characterized by pain and a pulling or popping feeling at the surgical site. Peritonitis presents with a rigid, boardlike abdomen, tenderness, and fever. The client's signs and symptoms aren't a normal reaction to surgery.

The nurse is conducting a community education program on peptic ulcer disease prevention. The nurse includes that the most common cause of peptic ulcers is: -stress and anxiety. -gram-negative bacteria. -alcohol and tobacco. -ibuprofen and aspirin.

-gram-negative bacteria. The nurse should include that the most common cause of peptic ulcers is gram-negative bacteria (Helicobacter pylori).

The nurse is caring for a patient who has been diagnosed with gastritis. To promote fluid balance when treating gastritis, the nurse knows that what minimal daily intake of fluids is required?

1.5 L

The nurse is caring for a patient who has been diagnosed with gastritis. To promote fluid balance when treating gastritis, the nurse knows that what minimal daily intake of fluids is required?

1.5 L Daily fluid intake and output are monitored to detect early signs of dehydration (minimal fluid intake of 1.5 L/day, minimal output of 0.5 mL/kg/h).

A client with peptic ulcer disease must begin triple medication therapy. For how long will the client follow this regimen?

10 to 14 days Recommended therapy for 10 to 14 days includes triple therapy with two antibiotics (eg, metronidazole [Flagyl] or amoxicillin [Amoxil] and clarithromycin [Biaxin]) plus a proton pump inhibitor (eg, lansoprazole [Prevacid], omeprazole [Prilosec], or rabeprazole [Aciphex]), or quadruple therapy with two antibiotics (metronidazole and tetracycline) plus a proton pump inhibitor and bismuth salts (Pepto-Bismol)

A client with peptic ulcer disease must begin triple medication therapy. For how long will the client follow this regimen? 15 to 20 days 10 to 14 days 4 to 6 days 7 to 9 days

10 to 14 days Explanation: Recommended therapy for 10 to 14 days includes triple therapy with two antibiotics (e.g., metronidazole [Flagyl] or amoxicillin [Amoxil] and clarithromycin [Biaxin]) plus a proton pump inhibitor (e.g., lansoprazole [Prevacid], omeprazole [Prilosec], or rabeprazole [Aciphex]), or quadruple therapy with two antibiotics (metronidazole and tetracycline) plus a proton pump inhibitor and bismuth salts (Pepto-Bismol).

26. A patient has been admitted to the hospital after diagnostic imaging revealed the presence of a gastric outlet obstruction (GOO). What is the nurses priority intervention? A) Administration of antiemetics B) Insertion of an NG tube for decompression C) Infusion of hypotonic IV solution D) Administration of proton pump inhibitors as ordered

Ans: B Feedback: In treating the patient with gastric outlet obstruction, the first consideration is to insert an NG tube to decompress the stomach. This is a priority over fluid or medication administration.

A client weighing 165 lb. (75 kg) is being treated for acute gastritis. Which amount of urine output for 4 hours would indicate an adequate fluid balance if the output should be 1 mL/kg/hour? _________________________________________

300 Explanation: To determine the client's weight in kg, divide the weight in lb. by 2.2 or 165 / 2.2 = 75 kg. Fluid balance for this client would be 75 mL/hr. For four hours, the client's output would need to be 300 mL as an indication of fluid balance.

A client weighs 215 lbs and is 5' 8" tall. The nurse would calculate this client's body mass index (BMI) as which of the following?

32.7

A client weighs 215 lbs and is 5' 8" tall. What would the nurse calculate this client's body mass index (BMI) as being?

32.7

A client weighs 215 lbs and is 5' 8" tall. The nurse would calculate this client's body mass index (BMI) as which of the following?

32.7 Using the formula for BMI, the client's weight in pounds (215) is divided by the height in inches squared (68 inches squared) and then multiplied by 703. The result would be 32.7.

A patient being treated for pyloric obstruction has a nasogastric (NG) tube in place to decompress the stomach. The nurse routinely checks for a residual amount that would indicate obstruction. Choose that amount.

450 mL A residual of greater than 400 mL strongly suggests obstruction.

The nurse inserts a nasogastric tube into the right nares of a patient. When testing the tube aspirate for pH to confirm placement, what does the nurse anticipate the pH will be if placement is in the lungs?

6

A client returns from the operating room after receiving extensive abdominal surgery. He has 1,000 mL of lactated Ringer's solution infusing via a central line. The physician orders the IV fluid to be infused at 125 mL/hr plus the total output of the previous hour. The drip factor of the tubing is 15 gtt/min, and the output for the previous hour was 75 mL via Foley catheter, 50 mL via nasogastric tube, and 10 mL via Jackson Pratt tube. For how many drops per mintue should the nurse set the IV flow rate to deliver the correct amount of fluid? Enter the correct number ONLY.

65 First, calculate the volume to be infused (in milliliters): 75 mL + 50 mL + 10 mL = 135 mL total output for the previous hour; 135 mL + 125 mL ordered as a constant flow = 260 mL to be infused over the next hour Next use the formula Volume to be infused/Total minutes to be infused x Drip factor = Drops per min In this case, 260 mL divided by 60 min x 15 gtt/min = 65 gtt/min

A client reports to the clinic, stating that she rapidly developed headache, abdominal pain, nausea, hiccuping, and fatigue about 2 hours ago. For dinner, she ate buffalo chicken wings and beer. Which of the following medical conditions is most consistent with the client's presenting problems?

Acute gastritis The client with acute gastritis may have a rapid onset of symptoms, including abdominal discomfort, headache, lassitude, nausea, anorexia, vomiting, and hiccuping, which can last from a few hours to a few days. Acute gastritis is often caused by dietary indiscretion--a person eats food that is irritating, too highly seasoned, or contaminated with disease-causing microorganisms.

A client comes to the clinic after developing a headache, abdominal pain, nausea, hiccupping, and fatigue about 2 hours ago. The client tells the nurse that the last food was buffalo chicken wings and beer. Which medical condition does the nurse find to be most consistent with the client's presenting problems? Duodenal ulcer Acute gastritis Gastric cancer Gastric ulcer

Acute gastritis Explanation: A client with acute gastritis may have a rapid onset of symptoms, including abdominal discomfort, headache, lassitude, nausea, anorexia, vomiting, and hiccupping, which can last from a few hours to a few days. Acute gastritis is often caused by dietary indiscretion-a person eats food that is irritating, too highly seasoned, or contaminated with disease-causing microorganisms. A client with a duodenal ulcer will present with heartburn, nausea, excessive gas and vomiting. A client with gastric cancer will have persistent symptoms of nausea and vomiting, not sudden symptoms. A client with a gastric ulcer will have bloating, nausea, and vomiting, but not necessarily hiccups.

The nurse practitioner suspects that a patient may have a gastric ulcer after completing a history and physical exam. Select an indicator that can be used to help establish the distinction between gastric and duodenal ulcers. Patient's age Sensitivity to the use of nonsteroidal anti-inflammatory drugs (NSAIDs) Presence of H. pylori Amount of hydrochloric acid (HCL) secretion in the stomach

Amount of hydrochloric acid (HCL) secretion in the stomach Explanation: A duodenal ulcer is characterized by hypersecretion of stomach acid, whereas a gastric ulcer shows evidence of hyposecretion of stomach acid. The other three choices have similar characteristics in both types of ulcers.

14. A patient was treated in the emergency department and critical care unit after ingesting bleach. What possible complication of the resulting gastritis should the nurse recognize? A) Esophageal or pyloric obstruction related to scarring B) Uncontrolled proliferation of H. pylori C) Gastric hyperacidity related to excessive gastrin secretion D) Chronic referred pain in the lower abdomen Test Bank - Brunner & Suddarth's Textbook of Medical-Surgical Nursing 14e (Hinkle 2017) 880

Ans: A Feedback: A severe form of acute gastritis is caused by the ingestion of strong acid or alkali, which may cause the mucosa to become gangrenous or to perforate. Scarring can occur, resulting in pyloric stenosis (narrowing or tightening) or obstruction. Chronic referred pain to the lower abdomen is a symptom of peptic ulcer disease, but would not be an expected finding for a patient who has ingested a corrosive substance. Bacterial proliferation and hyperacidity would not occur.

12. A patient presents to the walk-in clinic complaining of vomiting and burning in her mid-epigastria. The nurse knows that in the process of confirming peptic ulcer disease, the physician is likely to order a diagnostic test to detect the presence of what? A) Infection with Helicobacter pylori Test Bank - Brunner & Suddarth's Textbook of Medical-Surgical Nursing 14e (Hinkle 2017) 879 B) Excessive stomach acid secretion C) An incompetent pyloric sphincter D) A metabolic acidbase imbalance

Ans: A Feedback: H. pylori infection may be determined by endoscopy and histologic examination of a tissue specimen obtained by biopsy, or a rapid urease test of the biopsy specimen. Excessive stomach acid secretion leads to gastritis; however, peptic ulcers are caused by colonization of the stomach by H. pylori. Sphincter dysfunction and acidbase imbalances do not cause peptic ulcer disease.

2. A patient comes to the clinic complaining of pain in the epigastric region. What assessment question during the health interview would most help the nurse determine if the patient has a peptic ulcer? A) Does your pain resolve when you have something to eat? B) Do over-the-counter pain medications help your pain? C) Does your pain get worse if you get up and do some exercise? D) Do you find that your pain is worse when you need to have a bowel movement?

Ans: A Feedback: Pain relief after eating is associated with duodenal ulcers. The pain of peptic ulcers is generally unrelated to activity or bowel function and may or may not respond to analgesics.

28. Diagnostic imaging and physical assessment have revealed that a patient with peptic ulcer disease has suffered a perforated ulcer. The nurse recognizes that emergency interventions must be performed as soon as possible in order to prevent the development of what complication? A) Peritonitis B) Gastritis C) Gastroesophageal reflux D) Acute pancreatitis

Ans: A Feedback: Perforation is the erosion of the ulcer through the gastric serosa into the peritoneal cavity without warning. Chemical peritonitis develops within a few hours of perforation and is followed by bacterial peritonitis. Gastritis, reflux, and pancreatitis are not acute complications of a perforated ulcer.

8. A nurse in the postanesthesia care unit admits a patient following resection of a gastric tumor. Following immediate recovery, the patient should be placed in which position to facilitate patient comfort and gastric emptying? A) Fowlers B) Supine C) Left lateral D) Left Sims

Ans: A Feedback: Positioning the patient in a Fowlers position postoperatively promotes comfort and facilitates emptying of the stomach following gastric surgery. Any position that involves lying down delays stomach emptying and is not recommended for this type of patient. Supine positioning and the left lateral (left Sims) position do not achieve this goal.

3. A patient with a diagnosis of peptic ulcer disease has just been prescribed omeprazole (Prilosec). How should the nurse best describe this medications therapeutic action? A) This medication will reduce the amount of acid secreted in your stomach. Test Bank - Brunner & Suddarth's Textbook of Medical-Surgical Nursing 14e (Hinkle 2017) 875 B) This medication will make the lining of your stomach more resistant to damage. C) This medication will specifically address the pain that accompanies peptic ulcer disease. D) This medication will help your stomach lining to repair itself.

Ans: A Feedback: Proton pump inhibitors like Prilosec inhibit the synthesis of stomach acid. PPIs do not increase the durability of the stomach lining, relieve pain, or stimulate tissue repair.

29. A nurse is performing the admission assessment of a patient whose high body mass index (BMI) corresponds to class III obesity. In order to ensure empathic and patient-centered care, the nurse should do which of the following? A) Examine ones own attitudes towards obesity in general and the patient in particular. B) Dialogue with the patient about the lifestyle and psychosocial factors that resulted in obesity. C) Describe ones own struggles with weight gain and weight loss to the patient. D) Elicit the patients short-term and long-term goals for weight loss.

Ans: A Feedback: Studies suggest that health care providers, including nurses, harbor negative attitudes towards obese patients. Nurses have a responsibility to examine these attitudes and change them accordingly. This is foundational to all other areas of assessing this patient.

11. A patient who experienced an upper GI bleed due to gastritis has had the bleeding controlled and the patients condition is now stable. For the next several hours, the nurse caring for this patient should assess for what signs and symptoms of recurrence? A) Tachycardia, hypotension, and tachypnea B) Tarry, foul-smelling stools C) Diaphoresis and sudden onset of abdominal pain D) Sudden thirst, unrelieved by oral fluid administration

Ans: A Feedback: Tachycardia, hypotension, and tachypnea are signs of recurrent bleeding. Patients who have had one GI bleed are at risk for recurrence. Tarry stools are expected short-term findings after a hemorrhage. Hemorrhage is not normally associated with sudden thirst or diaphoresis.

38. A patient with gastritis required hospital treatment for an exacerbation of symptoms and receives a subsequent diagnosis of pernicious anemia due to malabsorption. When planning the patients continuing care in the home setting, what assessment question is most relevant? A) Does anyone in your family have experience at giving injections? B) Are you going to be anywhere with strong sunlight in the next few months? C) Are you aware of your blood type? D) Do any of your family members have training in first aid?

Ans: A Feedback: Test Bank - Brunner & Suddarth's Textbook of Medical-Surgical Nursing 14e (Hinkle 2017) 891 Patients with malabsorption of vitamin B12 need information about lifelong vitamin B12 injections; the nurse may instruct a family member or caregiver how to administer the injections or make arrangements for the patient to receive the injections from a health care provider. Questions addressing sun exposure, blood type and first aid are not directly relevant.

40. A patient has come to the clinic complaining of pain just above her umbilicus. When assessing the patient, the nurse notes Sister Mary Josephs nodules. The nurse should refer the patient to the primary care provider to be assessed for what health problem? A) A GI malignancy B) Dumping syndrome C) Peptic ulcer disease D) Esophageal/gastric obstruction

Ans: A Feedback: Test Bank - Brunner & Suddarth's Textbook of Medical-Surgical Nursing 14e (Hinkle 2017) 892 Palpable nodules around the umbilicus, called Sister Mary Josephs nodules, are a sign of a GI malignancy, usually a gastric cancer. This would not be a sign of dumping syndrome, peptic ulcer disease, or esophageal/gastric obstruction.

35. A patient has received a diagnosis of gastric cancer and is awaiting a surgical date. During the preoperative period, the patient should adopt what dietary guidelines? A) Eat small, frequent meals with high calorie and vitamin content. B) Eat frequent meals with an equal balance of fat, carbohydrates, and protein. C) Eat frequent, low-fat meals with high protein content. D) Try to maintain the pre-diagnosis pattern of eating.

Ans: A Feedback: The nurse encourages the patient to eat small, frequent portions of nonirritating foods to decrease gastric irritation. Food supplements should be high in calories, as well as vitamins A and C and iron, to enhance tissue repair.

16. A patient is one month postoperative following restrictive bariatric surgery. The patient tells the clinic nurse that he has been having trouble swallowing for the past few days. What recommendation should the nurse make? A) Eating more slowly and chewing food more thoroughly B) Taking an OTC antacid or drinking a glass of milk prior to each meal C) Chewing gum to cause relaxation of the lower esophageal sphincter D) Drinking at least 12 ounces of liquid with each meal

Ans: A Test Bank - Brunner & Suddarth's Textbook of Medical-Surgical Nursing 14e (Hinkle 2017) 881 Feedback: Dysphagia may be prevented by educating patients to eat slowly, to chew food thoroughly, and to avoid eating tough foods such as steak or dry chicken or doughy bread. After bariatric procedures, patients should normally not drink beverages with meals. Medications or chewing gum will not alleviate this problem.

39. A nurse is presenting a class at a bariatric clinic about the different types of surgical procedures offered by the clinic. When describing the implications of different types of surgeries, the nurse should address which of the following topics? Select all that apply. A) Specific lifestyle changes associated with each procedure B) Implications of each procedure for eating habits C) Effects of different surgeries on bowel function D) Effects of various bariatric surgeries on fertility E) Effects of different surgeries on safety of future immunizations

Ans: A, B, C Feedback: Different bariatric surgical procedures entail different lifestyle modifications; patients must be well informed about the specific lifestyle changes, eating habits, and bowel habits that may result from a particular procedure. Bariatric surgeries do not influence the future use of immunizations or fertility, though pregnancy should be avoided for 18 months after bariatric surgery.

1. A nurse is caring for a patient who just has been diagnosed with a peptic ulcer. When teaching the patient about his new diagnosis, how should the nurse best describe a peptic ulcer? A) Inflammation of the lining of the stomach B) Erosion of the lining of the stomach or intestine C) Bleeding from the mucosa in the stomach D) Viral invasion of the stomach wall

Ans: B Feedback: A peptic ulcer is erosion of the lining of the stomach or intestine. Peptic ulcers are often accompanied by bleeding and inflammation, but these are not the definitive characteristics.

6. A nurse caring for a patient who has had bariatric surgery is developing a teaching plan in anticipation of the patients discharge. Which of the following is essential to include? A) Drink a minimum of 12 ounces of fluid with each meal. B) Eat several small meals daily spaced at equal intervals. C) Choose foods that are high in simple carbohydrates. D) Sit upright when eating and for 30 minutes afterward.

Ans: B Feedback: Due to decreased stomach capacity, the patient must consume small meals at intervals to meet nutritional requirements while avoiding a feeling of fullness and complications such as dumping syndrome. The patient should not consume fluids with meals and low-Fowlers positioning is recommended during and after meals. Carbohydrates should be limited.

34. A patient has recently received a diagnosis of gastric cancer; the nurse is aware of the importance of assessing the patients level of anxiety. Which of the following actions is most likely to accomplish this? A) The nurse gauges the patients response to hypothetical outcomes. B) The patient is encouraged to express fears openly. Test Bank - Brunner & Suddarth's Textbook of Medical-Surgical Nursing 14e (Hinkle 2017) 889 C) The nurse provides detailed and accurate information about the disease. D) The nurse closely observes the patients body language.

Ans: B Feedback: Encouraging the patient to discuss his or her fears and anxieties is usually the best way to assess a patients anxiety. Presenting hypothetical situations is a surreptitious and possibly inaccurate way of assessing anxiety. Observing body language is part of assessment, but it is not the complete assessment. Presenting information may alleviate anxiety for some patients, but it is not an assessment.

33. A patient who is obese is exploring bariatric surgery options and presented to a bariatric clinic for preliminary investigation. The nurse interviews the patient, analyzing and documenting the data. Which of the following nursing diagnoses may be a contraindication for bariatric surgery? A) Disturbed Body Image Related to Obesity B) Deficient Knowledge Related to Risks and Expectations of Surgery C) Anxiety Related to Surgery D) Chronic Low Self-Esteem Related to Obesity

Ans: B Feedback: It is expected that patients seeking bariatric surgery may have challenges with body image and self- esteem related to their obesity. Anxiety is also expected when facing surgery. However, if the patients knowledge remains deficient regarding the risks and realistic expectations for surgery, this may show that the patient is not an appropriate surgical candidate.

10. A nurse is assessing a patient who has peptic ulcer disease. The patient requests more information about the typical causes of Helicobacter pylori infection. What would it be appropriate for the nurse to instruct the patient? A) Most affected patients acquired the infection during international travel. B) Infection typically occurs due to ingestion of contaminated food and water. C) Many people possess genetic factors causing a predisposition to H. pylori infection. D) The H. pylori microorganism is endemic in warm, moist climates.

Ans: B Feedback: Most peptic ulcers result from infection with the gram-negative bacteria H. pylori, which may be acquired through ingestion of food and water. The organism is endemic to all areas of the United States. Genetic factors have not been identified.

19. A patient comes to the bariatric clinic to obtain information about bariatric surgery. The nurse assesses the obese patient knowing that in addition to meeting the criterion of morbid obesity, a candidate for bariatric surgery must also demonstrate what? A) Knowledge of the causes of obesity and its associated risks B) Adequate understanding of required lifestyle changes C) Positive body image and high self-esteem D) Insight into why past weight loss efforts failed

Ans: B Feedback: Patients seeking bariatric surgery should be free of serious mental disorders and motivated to comply with lifestyle changes related to eating patterns, dietary choices, and elimination. While assessment of knowledge about causes of obesity and its associated risks as well as insight into the reasons why previous diets have been ineffective are included in the clients plan of care, these do not predict positive client outcomes following bariatric surgery. Most obese patients have an impaired body image and alteration in self-esteem. An obese patient with a positive body image would be unlikely to seek this surgery unless he or she was experiencing significant comorbidities.

27. A patient with a history of peptic ulcer disease has presented to the emergency department (ED) in distress. What assessment finding would lead the ED nurse to suspect that the patient has a perforated ulcer? A) The patient has abdominal bloating that developed rapidly. B) The patient has a rigid, boardlike abdomen that is tender. C) The patient is experiencing intense lower right quadrant pain. D) The patient is experiencing dizziness and confusion with no apparent hemodynamic changes.

Ans: B Feedback: Test Bank - Brunner & Suddarth's Textbook of Medical-Surgical Nursing 14e (Hinkle 2017) 886 An extremely tender and rigid (boardlike) abdomen is suggestive of a perforated ulcer. None of the other listed signs and symptoms is suggestive of a perforated ulcer.

23. A nurse is providing care for a patient who is postoperative day 2 following gastric surgery. The nurses assessment should be planned in light of the possibility of what potential complications? Select all that apply. A) Malignant hyperthermia Test Bank - Brunner & Suddarth's Textbook of Medical-Surgical Nursing 14e (Hinkle 2017) 884 B) Atelectasis C) Pneumonia D) Metabolic imbalances E) Chronic gastritis

Ans: B, C, D Feedback: After surgery, the nurse assesses the patient for complications secondary to the surgical intervention, such as pneumonia, atelectasis, or metabolic imbalances resulting from the GI disruption. Malignant hyperthermia is an intraoperative complication. Chronic gastritis is not a surgical complication.

5. A nurse is preparing to discharge a patient after recovery from gastric surgery. What is an appropriate discharge outcome for this patient? A) The patients bowel movements maintain a loose consistency. B) The patient is able to tolerate three large meals a day. C) The patient maintains or gains weight. D) The patient consumes a diet high in calcium. Test Bank - Brunner & Suddarth's Textbook of Medical-Surgical Nursing 14e (Hinkle 2017) 876

Ans: C Feedback: Expected outcomes for the patient following gastric surgery include ensuring that the patient is maintaining or gaining weight (patient should be weighed daily), experiencing no excessive diarrhea, and tolerating six small meals a day. Patients may require vitamin B12 supplementation by the intramuscular route and do not require a diet excessively rich in calcium.

17. A patient is receiving education about his upcoming Billroth I procedure (gastroduodenostomy). This patient should be informed that he may experience which of the following adverse effects associated with this procedure? A) Persistent feelings of hunger and thirst B) Constipation or bowel incontinence C) Diarrhea and feelings of fullness D) Gastric reflux and belching

Ans: C Feedback: Following a Billroth I, the patient may have problems with feelings of fullness, dumping syndrome, and diarrhea. Hunger and thirst, constipation, and gastric reflux are not adverse effects associated with this procedure.

20. A nurse is providing patient education for a patient with peptic ulcer disease secondary to chronic nonsteroidal anti-inflammatory drug (NSAID) use. The patient has recently been prescribed misoprostol (Cytotec). What would the nurse be most accurate in informing the patient about the drug? A) It reduces the stomachs volume of hydrochloric acid B) It increases the speed of gastric emptying C) It protects the stomachs lining D) It increases lower esophageal sphincter pressure

Ans: C Feedback: Misoprostol is a synthetic prostaglandin that, like prostaglandin, protects the gastric mucosa. NSAIDs decrease prostaglandin production and predispose the patient to peptic ulceration. Misoprostol does not reduce gastric acidity, improve emptying of the stomach, or increase lower esophageal sphincter pressure. Test Bank - Brunner & Suddarth's Textbook of Medical-Surgical Nursing 14e (Hinkle 2017) 883

The nurse is assessing a client with advanced gastric cancer. The nurse anticipates that the assessment will reveal which finding? Abdominal pain below the umbilicus Weight gain Increased appetite Bloating after meals

Bloating after meals Explanation: Symptoms of progressive disease include bloating after meals, weight loss, abdominal pain above the umbilicus, loss or decrease in appetite, and nausea or vomiting.

21. A nurse is providing anticipator guidance to a patient who is preparing for bariatric surgery. The nurse learns that the patient is anxious about numerous aspects of the surgery. What intervention is most appropriate to alleviate the patients anxiety? A) Emphasize the fact that bariatric surgery has a low risk of complications. B) Encourage the patient to focus on the benefits of the surgery. C) Facilitate the patients contact with a support group. D) Obtain an order for a PRN benzodiazepine.

Ans: C Feedback: Support groups can be highly beneficial in relieving preoperative and postoperative anxiety and in promoting healthy coping. This is preferable to antianxiety medications. Downplaying the risks of surgery or focusing solely on the benefits is a simplistic and patronizing approach.

7. A nurse is completing a health history on a patient whose diagnosis is chronic gastritis. Which of the data should the nurse consider most significantly related to the etiology of the patients health problem? A) Consumes one or more protein drinks daily. B) Takes over-the-counter antacids frequently throughout the day. C) Smokes one pack of cigarettes daily. D) Reports a history of social drinking on a weekly basis.

Ans: C Feedback: Test Bank - Brunner & Suddarth's Textbook of Medical-Surgical Nursing 14e (Hinkle 2017) 877 Nicotine reduces secretion of pancreatic bicarbonate, which inhibits neutralization of gastric acid and can underlie gastritis. Protein drinks do not result in gastric inflammation. Antacid use is a response to experiencing symptoms of gastritis, not the etiology of gastritis. Alcohol ingestion can lead to gastritis; however, this generally occurs in patients with a history of consumption of alcohol on a daily basis.

15. A patient who underwent gastric banding 3 days ago is having her diet progressed on a daily basis. Following her latest meal, the patient complains of dizziness and palpitations. Inspection reveals that the patient is diaphoretic. What is the nurses best action? A) Insert a nasogastric tube promptly. B) Reposition the patient supine. C) Monitor the patient closely for further signs of dumping syndrome. D) Assess the patient for signs and symptoms of aspiration.

Ans: C Feedback: The patients symptoms are characteristic of dumping syndrome, which results in a sensation of fullness, weakness, faintness, dizziness, palpitations, diaphoresis, cramping pains, and diarrhea. Aspiration is a less likely cause for the patients symptoms. Supine positioning will likely exacerbate the symptoms and insertion of an NG tube is contraindicated due to the nature of the patients surgery.

31. A patient who is obese has been unable to lose weight successfully using lifestyle modifications and has mentioned the possibility of using weight-loss medications. What should the nurse teach the patient about pharmacologic interventions for the treatment of obesity? A) Weight loss drugs have many side effects, and most doctors think theyll all be off the market in a few years. B) There used to be a lot of hope that medications would help people lose weight, but its been shown to be mostly a placebo effect. C) Medications can be helpful, but few people achieve and maintain their desired weight loss with medications alone. D) Medications are rapidly become the preferred method of weight loss in people for whom diet and exercise have not worked.

Ans: C Feedback: Though antiobesity drugs help some patients lose weight, their use rarely results in loss of more than 10% of total body weight. Patients are consequently unlikely to attain their desired weight through medication alone. They are not predicted to disappear from the market and results are not attributed to a placebo effect.

24. A patient is undergoing diagnostic testing for a tumor of the small intestine. What are the most likely symptoms that prompted the patient to first seek care? A) Hematemesis and persistent sensation of fullness B) Abdominal bloating and recurrent constipation C) Intermittent pain and bloody stool D) Unexplained bowel incontinence and fatty stools

Ans: C Feedback: When the patient is symptomatic from a tumor of the small intestine, benign tumors often present with intermittent pain. The next most common presentation is occult bleeding. The other listed signs and symptoms are not normally associated with the presentation of small intestinal tumors.

13. A patient with a peptic ulcer disease has had metronidazole (Flagyl) added to his current medication regimen. What health education related to this medication should the nurse provide? A) Take the medication on an empty stomach. B) Take up to one extra dose per day if stomach pain persists. C) Take at bedtime to mitigate the effects of drowsiness. D) Avoid drinking alcohol while taking the drug.

Ans: D Feedback: Alcohol must be avoided when taking Flagyl and the medication should be taken with food. This drug does not cause drowsiness and the dose should not be adjusted by the patient.

32. A patient has been diagnosed with peptic ulcer disease and the nurse is reviewing his prescribed medication regimen with him. What is currently the most commonly used drug regimen for peptic ulcers? Test Bank - Brunner & Suddarth's Textbook of Medical-Surgical Nursing 14e (Hinkle 2017) 888 A) Bismuth salts, antivirals, and histamine-2 (H2) antagonists B) H2 antagonists, antibiotics, and bicarbonate salts C) Bicarbonate salts, antibiotics, and ZES D) Antibiotics, proton pump inhibitors, and bismuth salts

Ans: D Feedback: Currently, the most commonly used therapy for peptic ulcers is a combination of antibiotics, proton pump inhibitors, and bismuth salts that suppress or eradicate H. pylori. H2 receptor antagonists are used to treat NSAID-induced ulcers and other ulcers not associated with H. pylori infection, but they are not the drug of choice. Bicarbonate salts are not used. ZES is the Zollinger-Ellison syndrome and not a drug.

22. A patient has just been diagnosed with acute gastritis after presenting in distress to the emergency department with abdominal symptoms. What would be the nursing care most needed by the patient at this time? A) Teaching the patient about necessary nutritional modification B) Helping the patient weigh treatment options C) Teaching the patient about the etiology of gastritis D) Providing the patient with physical and emotional support

Ans: D Feedback: For acute gastritis, the nurse provides physical and emotional support and helps the patient manage the symptoms, which may include nausea, vomiting, heartburn, and fatigue. The scenario describes a newly diagnosed patient; teaching about the etiology of the disease, lifestyle modifications, or various treatment options would be best provided at a later time.

A patient comes to the clinic with the complaint, "I think I have an ulcer." What is a characteristic associated with peptic ulcer pain that the nurse should inquire about? Select all that apply.

Burning sensation localized in the back or mid-epigastrium Feeling of emptiness that precedes meals from 1 to 3 hours Severe gnawing pain that increases in severity as the day progresses

Which of the following clients is at highest risk for peptic ulcer disease?

Client with blood type O

18. A patient has experienced symptoms of dumping syndrome following bariatric surgery. To what physiologic phenomenon does the nurse attribute this syndrome? A) Irritation of the phrenic nerve due to diaphragmatic pressure B) Chronic malabsorption of iron and vitamins A and C C) Reflux of bile into the distal esophagus D) A sudden release of peptides

Ans: D Feedback: For many years, it had been theorized that the hypertonic gastric food boluses that quickly transit into the intestines drew extracellular fluid from the circulating blood volume into the small intestines to dilute the high concentration of electrolytes and sugars, resulting in symptoms. Now, it is thought that this rapid transit of the food bolus from the stomach into the small intestines instead causes a rapid and Test Bank - Brunner & Suddarth's Textbook of Medical-Surgical Nursing 14e (Hinkle 2017) 882 exuberant release of metabolic peptides that are responsible for the symptoms of dumping syndrome. It is not a result of phrenic nerve irritation, malabsorption, or bile reflux.

37. A nurse is caring for a patient hospitalized with an exacerbation of chronic gastritis. What health promotion topic should the nurse emphasize? A) Strategies for maintaining an alkaline gastric environment B) Safe technique for self-suctioning C) Techniques for positioning correctly to promote gastric healing D) Strategies for avoiding irritating foods and beverages

Ans: D Feedback: Measures to help relieve pain include instructing the patient to avoid foods and beverages that may be irritating to the gastric mucosa and instructing the patient about the correct use of medications to relieve chronic gastritis. An alkaline gastric environment is neither possible nor desirable. There is no plausible need for self-suctioning. Positioning does not have a significant effect on the presence or absence of gastric healing.

9. A community health nurse is preparing for an initial home visit to a patient discharged following a total gastrectomy for treatment of gastric cancer. What would the nurse anticipate that the plan of care is most likely to include? A) Enteral feeding via gastrostomy tube (G tube) B) Gastrointestinal decompression by nasogastric tube C) Periodic assessment for esophageal distension D) Monthly administration of injections of vitamin B12

Ans: D Feedback: Since vitamin B12 is absorbed in the stomach, the patient requires vitamin B12 replacement to prevent pernicious anemia. A gastrectomy precludes the use of a G tube. Since the stomach is absent, a nasogastric tube would not be indicated. As well, this is not possible in the home setting. Since there is no stomach to act as a reservoir and fluids and nutrients are passing directly into the jejunum, distension Test Bank - Brunner & Suddarth's Textbook of Medical-Surgical Nursing 14e (Hinkle 2017) 878 is unlikely.

4. A nurse is admitting a patient diagnosed with late-stage gastric cancer. The patients family is distraught and angry that she was not diagnosed earlier in the course of her disease. What factor contributes to the fact that gastric cancer is often detected at a later stage? A) Gastric cancer does not cause signs or symptoms until metastasis has occurred. B) Adherence to screening recommendations for gastric cancer is exceptionally low. C) Early symptoms of gastric cancer are usually attributed to constipation. D) The early symptoms of gastric cancer are usually not alarming or highly unusual.

Ans: D Feedback: Symptoms of early gastric cancer, such as pain relieved by antacids, resemble those of benign ulcers and are seldom definitive. Symptoms are rarely a cause for alarm or for detailed diagnostic testing. Symptoms precede metastasis, however, and do not include constipation.

36. A nurse is caring for a patient who has a diagnosis of GI bleed. During shift assessment, the nurse finds the patient to betachycardic and hypotensive, and the patient has an episode of hematemesis while the nurse is in the room. In addition to monitoring the patients vital signs and level of conscious, what would be a priority nursing action for this patient? A) Place the patient in a prone position. B) Provide the patient with ice water to slow any GI bleeding. C) Prepare for the insertion of an NG tube. D) Notify the physician. Test Bank - Brunner & Suddarth's Textbook of Medical-Surgical Nursing 14e (Hinkle 2017) 890

Ans: D Feedback: The nurse must always be alert for any indicators of hemorrhagic gastritis, which include hematemesis (vomiting of blood), tachycardia, and hypotension. If these occur, the physician is notified and the patients vital signs are monitored as the patients condition warrants. Putting the patient in a prone position could lead to aspiration. Giving ice water is contraindicated as it would stimulate more vomiting.

25. A patient is recovering in the hospital following gastrectomy. The nurse notes that the patient has become increasingly difficult to engage and has had several angry outbursts at various staff members in recent days. The nurses attempts at therapeutic dialogue have been rebuffed. What is the nurses most appropriate action? A) Ask the patients primary care provider to liaise between the nurse and the patient. B) Delegate care of the patient to a colleague. C) Limit contact with the patient in order to provide privacy. Test Bank - Brunner & Suddarth's Textbook of Medical-Surgical Nursing 14e (Hinkle 2017) 885 D) Make appropriate referrals to services that provide psychosocial support.

Ans: D Feedback: The nurse should enlist the services of clergy, psychiatric clinical nurse specialists, psychologists, social workers, and psychiatrists, if needed. This is preferable to delegating care, since the patient has become angry with other care providers as well. It is impractical and inappropriate to expect the primary care provider to act as a liaison. It would be inappropriate and unsafe to simply limit contact with the patient.

A client with gastric cancer is scheduled to undergo a Billroth II procedure. The client's spouse asks how much of the client's stomach will be removed. Which of the following would be the most accurate response from the nurse?

Approximately 75%

While caring for a patient who has had radical neck surgery, the nurse notices an abnormal amount of serosanguineous secretions in the wound suction unit during the first postoperative day. What does the nurse know is an expected amount of drainage in the wound unit?

Approximately 80 to 120 mL

A client has recently been diagnosed with gastric cancer. He has a history of tobacco use and was diagnosed 10 years ago with pernicious anemia. He and his family are shocked about the possibility of this diagnosis because he has been asymptomatic prior to recent complaints of pain and multiple gastrointestinal symptoms. On palpation, the nurse notes two signs that confirm metastasis to the liver. Which of the following are signs? Choose all that apply.

Ascites Hepatomegaly

A client has recently been diagnosed with gastric cancer. On palpation, the nurse would note what two signs that confirm metastasis to the liver? Select all that apply. Petechiae at the palpation site Distended bladder Hepatomegaly Sister Mary Joseph's nodules Ascites

Ascites Hepatomegaly Explanation: The physical examination is usually not helpful in detecting the cancer because most early gastric tumors are not palpable. Advanced gastric cancer may be palpable as a mass. Ascites and hepatomegaly (enlarged liver) may be apparent if the cancer cells have metastasized to the liver. Palpable nodules around the umbilicus, called Sister Mary Joseph's nodules, are a sign of a GI malignancy, usually a gastric cancer. A distended bladder is not significant. Petechiae at the palpation site is a distractor for the question.

A client who underwent abdominal surgery and has a nasogastric (NG) tube in place begins to complain of abdominal pain that he describes as "feeling full and uncomfortable." Which assessment should the nurse perform first?

Assess patency of the NG tube.

When caring for a client with an acute exacerbation of a peptic ulcer, the nurse finds the client doubled up in bed with severe pain in the right shoulder. What is the initial appropriate action by the nurse?

Assess the client's abdomen and vital signs.

When caring for a client with an acute exacerbation of a peptic ulcer, the nurse finds the client doubled up in bed with severe pain to his right shoulder. The intial appropriate action by the nurse is to

Assess the client's abdomen and vital signs. Signs and symptoms of perforation includes sudden, severe upper abdominal pain (persisting and increasing in intensity); pain may be referred to the shoulders, especially the right shoulder, because of irritation of the phrenic nerve in the diaphragm. The nurse should assess the vital signs and abdomen prior to notifying the physician. Irrigation of the NG tube should not be performed because the additional fluid may be spilled into the peritoneal cavity, and the client should be placed in a position of comfort, usually on the side with the head slightly elevated.

A young adult client is prescribed misoprostol to prevent gastric ulcers caused by frequent use of nonsteroidal anti-inflammatory agents for an autoimmune disorder. For which reason will the nurse question giving the client a dose of this medication? Awaiting the results of a pregnancy test Needs to be taken without food May cause diarrhea and cramping Can cause constipation

Awaiting the results of a pregnancy test Explanation: Misoprostol is a synthetic prostaglandin that protects the gastric mucosa from agents that cause ulcers, and also increases mucus production and bicarbonate levels. It is a pregnancy category X medication and should not be taken by a pregnant client as it can soften the cervix and result in miscarriage or premature labor. This medication does not cause constipation. Sucralfate needs to be taken without food. Misoprostol can cause diarrhea and cramping; however, this is not the reason to question giving the client a dose of the medication.

A client is admitted to the health care facility with a diagnosis of a bleeding gastric ulcer. The nurse expects the client's stools to have which description?

Black and tarry

A client is admitted to the health care facility with a diagnosis of a bleeding gastric ulcer. The nurse expects the client's stools to have which description? Clay-colored Coffee-ground-like Black and tarry Bright red

Black and tarry Explanation: Black, tarry stools are a sign of bleeding high in the GI tract, as from a gastric ulcer, and result from the action of digestive enzymes on the blood. Vomitus associated with upper GI tract bleeding commonly is described as coffee-ground-like. Clay-colored stools are associated with biliary obstruction. Bright red stools indicate lower GI tract bleeding.

The nurse is assessing a client with a bleeding gastric ulcer. When examining the client's stool, which characteristic would the nurse be most likely to find?

Black and tarry appearance

The nurse is assessing a client with a bleeding gastric ulcer. When examining the client's stool, which of the following characteristics would the nurse be most likely to find?

Black and tarry appearance

A client with peptic ulcer disease has a blood pressure of 88/40 mm Hg, dizziness, and nausea. Which complication will the nurse suspect is occurring with this client?

Bleeding from the ulcer

A client with peptic ulcer disease has a blood pressure of 88/40 mm Hg, dizziness, and nausea. Which complication will the nurse suspect is occurring with this client? Medication adverse effects Bleeding from the ulcer Evidence of ulcer healing Allergic response to the medication

Bleeding from the ulcer Explanation: Bleeding peptic ulcers account for 27% to 40% of all upper GI bleeds and it may be manifested by hematemesis or melena. Faintness or dizziness and nausea may precede or accompany bleeding. A low blood pressure could indicate active bleeding. The client's symptoms do not indicate evidence of the ulcer healing. It is unlikely that the client's symptoms are adverse effects or allergic responses to the medications.

A client comes to the clinic after developing a headache, abdominal pain, nausea, hiccupping, and fatigue about 2 hours ago. The client tells the nurse that the last food was buffalo chicken wings and beer. Which medical condition does the nurse find to be most consistent with the client's presenting problems? - Acute gastritis - Duodenal ulcer - Gastric cancer - Gastric ulcer

Correct response: Acute gastritis Explanation: A client with acute gastritis may have a rapid onset of symptoms, including abdominal discomfort, headache, lassitude, nausea, anorexia, vomiting, and hiccupping, which can last from a few hours to a few days. Acute gastritis is often caused by dietary indiscretion-a person eats food that is irritating, too highly seasoned, or contaminated with disease-causing microorganisms. A client with a duodenal ulcer will present with heartburn, nausea, excessive gas and vomiting. A client with gastric cancer will have persistent symptoms of nausea and vomiting, not sudden symptoms. A client with a gastric ulcer will have bloating, nausea, and vomiting, but not necessarily hiccups.

The nurse is assessing an 80-year-old client for signs and symptoms of gastric cancer. The nurse differentiates which as a sign/symptom of gastric cancer in the geriatric client, but not in a client under the age of 75? - Abdominal mass - Agitation - Hepatomegaly - Ascites

Correct response: Agitation Explanation: The nurse understands that agitation, along with confusion and restlessness, may be the only signs/symptoms seen of gastric cancer in the older client. Abdominal mass, hepatomegaly, and ascites may all be signs/symptoms of advanced gastric cancer.

The nurse practitioner suspects that a patient may have a gastric ulcer after completing a history and physical exam. Select an indicator that can be used to help establish the distinction. - Amount of hydrochloric acid (HCL) secretion in the stomach - Sensitivity to the use of nonsteroidal anti-inflammatory drugs (NSAIDs) - Presence of H. pylori - Patient's age

Correct response: Amount of hydrochloric acid (HCL) secretion in the stomach Explanation: A duodenal ulcer is characterized by hypersecretion of stomach acid, whereas a gastric ulcer evidences hyposecretion of stomach acid. The other three choices have similar characteristics in both types of ulcers.

A client is scheduled for removal of the lower portion of the antrum of the stomach and a small portion of the duodenum and pylorus. What surgical procedure will the nurse prepare the client for? - Vagotomy - Pyloroplasty - Billroth I - Billroth II

Correct response: Billroth I Explanation: A Billroth I is the removal of the lower portion (antrum) of the stomach (which contains the cells that secrete gastrin) as well as a small portion of the duodenum and pylorus. A vagotomy is a surgical dissection of the vagus nerve to decrease gastric acid. A pyloroplasty is a procedure to widen the pylorus. A Billroth II is the removal of the lower portion (antrum) of stomach with anastomosis to the jejunum.

A patient comes to the clinic with the complaint, "I think I have an ulcer." What is a characteristic associated with peptic ulcer pain that the nurse should inquire about? Select all that apply. - Burning sensation localized in the back or mid-epigastrium - Feeling of emptiness that precedes meals from 1 to 3 hours - Severe gnawing pain that increases in severity as the day progresses - Pain that radiates to the shoulder or jaw - Vomiting without associated nausea

Correct response: Burning sensation localized in the back or mid-epigastrium Feeling of emptiness that precedes meals from 1 to 3 hours Severe gnawing pain that increases in severity as the day progresses Explanation: As a rule, the patient with an ulcer complains of dull, gnawing pain or a burning sensation in the mid-epigastrium or the back. Although vomiting is rare in uncomplicated peptic ulcer, it may be a symptom of a complication of an ulcer.

A client has given a confirmed diagnosis of gastric cancer. Two more procedures may be performed to assess tumor depth and lymph node involvement and surgical respectability. Which two are the procedures? Choose the two that apply. - Barium x-ray of the upper GI tract - Esophagogastroduodenoscopy (EGD) - Computed tomography (CT) - Endoscopic ultrasound

Correct response: Computed tomography (CT) Endoscopic ultrasound Explanation: Esophagogastroduodenoscopy for biopsy and cytologic washings is the diagnostic study of choice, and a barium x-ray examination of the upper GI tract may also be performed. Endoscopic ultrasound is an important tool to assess tumor depth and any lymph node involvement. Pelvic ultrasound is not used to confirm the diagnosis of gastric cancer.

A morbidly obese client asks the nurse if medications are available to assist with weight loss. The nurse knows that the client would not be a candidate for phentermine if the following is part of the client's health history: - Coronary artery disease - Diabetes - Use of lithium - Peptic ulcer disease

Correct response: Coronary artery disease Explanation: Phentermine, which requires a prescription, stimulates central noradrenergic receptors, causing appetite suppression. It may increase blood pressure and should not be taken by people with a history of heart disease, uncontrolled hypertension, hyperthyroidism, or glaucoma.

A client with gastric cancer is having a resection. What is the nursing management priority for this client? - Discharge planning - Correcting nutritional deficits - Preventing deep vein thrombosis (DVT) - Teaching about radiation treatment

Correct response: Correcting nutritional deficits Explanation: Clients with gastric cancer commonly have nutritional deficits and may have cachexia. Therefore, correcting nutritional deficits is a top priority. Discharge planning before surgery is important, but correcting the nutritional deficits is a higher priority. Radiation therapy hasn't been proven effective for gastric cancer, and teaching about it preoperatively wouldn't be appropriate. Preventing DVT isn't a high priority before surgery, but it assumes greater importance after surgery.

A client with acute gastritis asks the nurse what might have caused the problem. What is a possible cause of acute gastritis? - Dietary indiscretion - Overuse of allergy medicine - Excessive alcohol intake - Drinking fruit juices - Radiation therapy

Correct response: Dietary indiscretion Excessive alcohol intake Radiation therapy Explanation: Possible causes of gastritis include dietary indiscretion, overuse of aspirin and other nonsteroidal anti-inflammatory drugs, excessive alcohol intake, bile reflux, and radiation therapy. Allergy medicine and fruit juices are not causes of acute gastritis.

A patient comes to the clinic complaining of pain in the epigastric region. The nurse suspects that the patient's pain is related to a peptic ulcer when the patient states the pain is relieved by what? - Eating - Drinking milk - Suppressing emesis - Having a bowel movement

Correct response: Eating Explanation: Taking antacids, eating, or vomiting often relieves the pain. Pain occurs about 2 hours after eating. Milk is contraindicated in relieving peptic ulcer pain.

The nurse is developing a plan of care for a patient with peptic ulcer disease. What nursing interventions should be included in the care plan? Select all that apply. - Making neurovascular checks every 4 hours - Frequently monitoring hemoglobin and hematocrit levels - Observing stools and vomitus for color, consistency, and volume - Checking the blood pressure and pulse rate every 15 to 20 minutes - Inserting an indwelling catheter for incontinence

Correct response: Frequently monitoring hemoglobin and hematocrit levels Observing stools and vomitus for color, consistency, and volume Checking the blood pressure and pulse rate every 15 to 20 minutes Explanation: The nurse assesses the patient for faintness or dizziness and nausea, which may precede or accompany bleeding. The nurse must monitor vital signs frequently and evaluate the patient for tachycardia, hypotension, and tachypnea. Other nursing interventions include monitoring the hemoglobin and hematocrit, testing the stool for gross or occult blood, and recording hourly urinary output to detect anuria or oliguria (absence of or decreased urine production).

Which diagnostic test would be used first to evaluate a client with upper GI bleeding? Upper GI series Endoscopy Hemoglobin and hematocrit Arteriography

Correct response: Hemoglobin and hematocrit Explanation: The nurse assesses for faintness or dizziness and nausea, which may precede or accompany bleeding. It is important to monitor vital signs frequently and to evaluate for tachycardia, hypotension, and tachypnea. Other nursing interventions include monitoring the hemoglobin and hematocrit, testing the stool for gross or occult blood, and recording hourly urinary output to detect anuria or oliguria (absence of or decreased urine production). If bleeding cannot be managed by the measures described, other treatment modalities such as endoscopy may be used to halt bleeding and avoid surgical intervention. There is debate regarding how soon endoscopy should be performed. Some clinicians believe endoscopy should be performed within the first 24 hours after hemorrhaging has ceased. Others believe endoscopy may be performed during acute bleeding, as long as the esophageal or gastric area can be visualized (blood may decrease visibility). An upper GI is less accurate than endoscopy and would not reveal a bleed. Arteriography is an invasive study associated with life-threatening complications and would not be used for an initial evaluation.

The nurse is cautiously assessing a client admitted with peptic ulcer disease because the most common complication that occurs in 10% to 20% of clients is: - Hemorrhage - Intractable ulcer - Perforation - Pyloric obstruction

Correct response: Hemorrhage Explanation: Hemorrhage, the most common complication, occurs in 10% to 20% of clients with peptic ulcers. Bleeding may be manifested by hematemesis or melena. Perforation is erosion of the ulcer through the gastric serosa into the peritoneal cavity without warning. Intractable ulcer refers to one that is hard to treat, relieve, or cure. Pyloric obstruction, also called gastric outlet obstruction (GOO), occurs when the area distal to the pyloric sphincter becomes scarred and stenosed from spasm or edema or from scar tissue that forms when an ulcer alternately heals and breaks down.

Which of the following is the most common complication associated with peptic ulcer? - Hemorrhage - Vomiting - Elevated temperature - Abdominal pain

Correct response: Hemorrhage Explanation: Hemorrhage, the most common complication, occurs in 28% to 59% of patients with peptic ulcers. Vomiting, elevated temperature, and abdominal pain are not the most common complications of a peptic ulcer.

The nurse is caring for a client who has just returned from the PACU after surgery for peptic ulcer disease. For what potential complications does the nurse know to monitor? Select all that apply. - Hemorrhage - Inability to clear secretions - Perforation - Penetration - Pyloric obstruction - Cachexia

Correct response: Hemorrhage Perforation Penetration Pyloric obstruction Explanation: Potential complications may include hemorrhage, perforation, penetration, and pyloric obstruction. A client who has had surgery for peptic ulcer disease may have a decreased appetite in the immediate postoperative stage, but it is not something the nurse would monitor for and would not cause cachexia. Inability to clear secretions is generally not a complication of peptic ulcer surgery.

Which of the following dietary guidelines should be followed following bariatric surgery? Select all that apply. - Include two protein snacks per day. - Eat slowly. - Eat three meals per day. - Eat and drink at the same time. - Total meal size should be 1 cup.

Correct response: Include two protein snacks per day. Eat slowly. Eat three meals per day. Explanation: Dietary guidelines for the patient who has had bariatric surgery include eating slowly, eating three meals per day, and including two protein snacks per day. The patient should avoid eating and drinking at the same time and his or her total meal size should be less than 1 cup.

A client undergoes total gastrectomy. Several hours after surgery, the nurse notes that the client's nasogastric (NG) tube has stopped draining. How should the nurse respond? - Notify the health care provider. - Reposition the tube. - Irrigate the tube. - Increase the suction level.

Correct response: Notify the health care provider. Explanation: The nurse should notify the health care provider because an NG tube that fails to drain during the postoperative period may be clogged, which could increase pressure on the suture site because fluid isn't draining adequately. Repositioning or irrigating an NG tube in a client who has undergone gastric surgery can disrupt the anastomosis. Increasing the level of suction may cause trauma to GI mucosa or the suture line.

Peptic ulcer disease occurs more frequently in people with which blood type? - A - B - AB - O

Correct response: O Explanation: People with blood type O are more susceptible to peptic ulcers than those with blood type A, B, or AB.

A patient asks the home health nurse from what the distressing symptoms of dumping syndrome result. What physiological occurrence should the nurse explain? - Irritation of the phrenic nerve due to diaphragmatic pressure - Chronic malabsorption of iron and vitamins A and C - Reflux of bile into the distal esophagus - Osmotic transport of extracellular fluid into the gastrointestinal tract

Correct response: Osmotic transport of extracellular fluid into the gastrointestinal tract Explanation: Following gastric surgery, the gastric remnant is anastomosed to the jejunum. When substances high in carbohydrates and electrolytes are ingested rapidly, they pass directly into the jejunum. Extracellular fluid from the bloodstream is drawn into the jejunum to dilute these hypertonic intestinal contents. Irritation of the phrenic nerve causes hiccups. Reflux of bile is an etiologic factor associated with the development of gastroesophageal reflux disease (GERD). Patients who have undergone partial gastrectomy or bariatric surgery may absorb vitamins and minerals less effectively; however, this change is unrelated to the occurrence of dumping syndrome.

Which of the following is considered an early symptom of gastric cancer? - Pain relieved by antacids - Weight loss - Bloating after meals - Dyspepsia

Correct response: Pain relieved by antacids Explanation: Symptoms of early disease, such as pain relieved by antacids, resemble those of benign ulcers and are seldom definitive. Symptoms of progressive disease include weight loss, bloating after meals, and dyspepsia.

Which of the following manifestations are associated with a deficiency of vitamin B12? Select all that apply. - Pernicious anemia - Macrocytic anemia - Thrombocytopenia - Loss of hair - Lethargy

Correct response: Pernicious anemia Macrocytic anemia Thrombocytopenia Explanation: Decreased vitamin B12 can result in pernicious anemia, macrocytic anemia, and thrombocytopenia. Decreased iron can result in lethargy and loss of hair.

The nurse is teaching a client with peptic ulcer disease who has been prescribed misoprostol. What information from the nurse would be most accurate about misoprostol? - Works best when taken on an empty stomach - Increases the speed of gastric emptying - Prevents ulceration in clients taking nonsteroidal anti-inflammatory drugs (NSAIDs) - Decreases mucus production

Correct response: Prevents ulceration in clients taking nonsteroidal anti-inflammatory drugs (NSAIDs) Explanation: Misoprostol (Cytotec) is a synthetic prostaglandin that protects the gastric mucosa against ulceration and is used in clients who take NSAIDs. Misoprostol should be taken with food. It does not improve emptying of the stomach, and it increases (not decreases) mucus production.

The nurse advises the patient who has just been diagnosed with acute gastritis to: - Take an emetic to rid the stomach of the irritating products. - Refrain from food until the GI symptoms subside. - Restrict food and fluids for 12 hours. - Restrict all food for 72 hours to rest the stomach.

Correct response: Refrain from food until the GI symptoms subside. Explanation: It usually takes 24 to 48 hours for the stomach to recover from an attack. Refraining from food until symptoms subside is recommended, but liquids should be taken in moderation. Emetics and vomiting can cause damage to the esophagus.

A health care provider counsels a client about bariatric surgery and recommends the Roux-en-Y gastric bypass. What is the best response by the nurse to further explain this procedure to the client? - Gastroplasty with a vertical band allowing for a pouch with a 15 to 20 mL capacity - Biliopancreatic diversion with a duodenal switch - Separation of the jejunum with an anastomosis - Gastric banding that incorporates a prosthetic device to restrict oral intake

Correct response: Separation of the jejunum with an anastomosis Explanation: The Roux-en-Y gastric bypass is recommended for long-term weight loss because it uses a combined restrictive and malabsorptive procedure.

A health care provider counsels a client about bariatric surgery and recommends the Roux-en-Y gastric bypass. What is the best response by the nurse to further explain this procedure to the client? Gastroplasty with a vertical band allowing for a pouch with a 15 to 20 mL capacity Biliopancreatic diversion with a duodenal switch Separation of the jejunum with an anastomosis Gastric banding that incorporates a prosthetic device to restrict oral intake

Correct response: Separation of the jejunum with an anastomosis Explanation: The Roux-en-Y gastric bypass is recommended for long-term weight loss because it uses a combined restrictive and malabsorptive procedure.

A client is recovering from gastric surgery. Toward what goal should the nurse progress the client's enteral intake? Three meals and 120 ml fluid daily Three meals and three snacks and 120 mL fluid daily Six small meals and 120 mL fluid daily Six small meals daily with 120 mL fluid between meals

Correct response: Six small meals daily with 120 mL fluid between meals Explanation: After the return of bowel sounds and removal of the nasogastric tube, the nurse may give fluids, followed by food in small portions. Foods are gradually added until the client can eat six small meals a day and drink 120 mL of fluid between meals.

A nurse is monitoring a client with peptic ulcer disease. Which assessment findings would most likely indicate perforation of the ulcer? Select all that apply. - Tachycardia - Hypotension - Mild epigastric pain - A rigid, board-like abdomen - Diarrhea

Correct response: Tachycardia Hypotension A rigid, board-like abdomen Explanation: Signs and symptoms of perforation include sudden, severe upper abdominal pain (persisting and increasing in intensity); pain, which may be referred to the shoulders, especially the right shoulder, because of irritation of the phrenic nerve in the diaphragm; vomiting; collapse (fainting); extremely tender and rigid (board-like) abdomen; and hypotension and tachycardia, indicating shock. Perforation is a surgical emergency.

A client taking metronidazole for the treatment of H. pylori states that the medication is causing nausea. What teaching should the nurse provide to the client to alleviate the nausea? - Discontinue the use of the medication. - Ask the healthcare provider to prescribe another type of antibiotic. - Take the medication with meals to decrease the nausea. - Crush the medication and put it in applesauce.

Correct response: Take the medication with meals to decrease the nausea. Explanation: Metronidazole (Flagyl) should be administered with meals to decrease GI upset. The client should not stop the medication without discussing it with a prescribing healthcare provider. Crushing the medication will not help the nausea because it is the same medication.

A healthcare provider prescribes a combination of three drugs to treat reoccurring peptic ulcer disease, and the client asks the nurse the reason for all the medications. What teaching should the nurse review with the client? - The antibiotics, prostaglandin E1 analogs, and bismuth salts will work together to suppress or eradicate H. pylori. - The proton pump inhibitors, prostaglandin E1 analogs, and bismuth salts will suppress or eradicate H. pylori. - The bismuth salts, antibiotics, and proton pump inhibitors will work together to suppress or eradicate H. pylori. - The prostaglandin E1 analogs, antibiotics, and proton pump inhibitors will work together to suppress or eradicate H. pylori.

Correct response: The bismuth salts, antibiotics, and proton pump inhibitors will work together to suppress or eradicate H. pylori. Explanation: The recommended triple combination of bismuth salts, antibiotics, and proton pump inhibitors will suppress or eradicate H. pylori. Prostaglandin E1 analogs enhance mucosal resistance to injury; they do not suppress or eradicate H. pylori.

Which of the following ulcers is associated with extensive burn injury?

Curling's ulcer Curling's ulcer is frequently observed about 72 hours after extensive burns and involves the antrum of the stomach or the duodenum.

Which is an accurate statement regarding gastric cancer? - The incidence of stomach cancer continues to decrease in the United States. - Most gastric cancer-related deaths occur in people younger than 40 years. - Females have a higher incidence of gastric cancers than males. - A diet high in smoked foods and low in fruits and vegetables may decrease the risk of gastric cancer.

Correct response: The incidence of stomach cancer continues to decrease in the United States. Explanation: While the incidence in the United States continues to decrease, gastric cancer still accounts for 10,700 deaths annually. While gastric cancer deaths occasionally occur in younger people, most occur in people older than 40 years of age. Males have a higher incidence of gastric cancers than females. More accurately, a diet high in smoked foods and low in fruits and vegetables may increase the risk of gastric cancer.

Which of the following interventions are appropriate for clients with gastritis? Select all that apply. - Use a calm approach to reduce anxiety. - Give the client food and fluids every 4 hours. - Discourage cigarette smoking. - Notify the physician of indicators of hemorrhagic gastritis. - Provide general education about how to prevent recurrences.

Correct response: Use a calm approach to reduce anxiety. Discourage cigarette smoking. Notify the physician of indicators of hemorrhagic gastritis. Explanation: The nurse should use a calm approach when answering questions and providing teaching. He or she should discuss smoking cessation and monitor for any indicators of hemorrhagic gastritis. The client will take nothing by mouth for up to a few days until symptoms subside. The nurse needs to develop an individualized teaching plan for the client that includes information about stress management, diet, and medications.

Which statement correctly identifies a difference between duodenal and gastric ulcers? - Malignancy is associated with duodenal ulcer. - Weight gain may occur with a gastric ulcer. - A gastric ulcer is caused by hypersecretion of stomach acid. - Vomiting is uncommon in clients with duodenal ulcers.

Correct response: Vomiting is uncommon in clients with duodenal ulcers. Explanation: Vomiting is uncommon in clients diagnosed with duodenal ulcer. Malignancy is associated with a gastric ulcer. Weight gain may occur with a duodenal ulcer. Duodenal ulcers cause hypersecretion of stomach acid.

The nurse is creating a discharge plan of care for a client with a peptic ulcer. The nurse tells the client to avoid acetaminophen. decaffeinated coffee. skim milk. octreotide.

Correct response: decaffeinated coffee. Explanation: The nurse should include avoidance of decaffeinated coffee in the client's discharge teaching plan. Decaffeinated coffee is avoided to keep from overstimulating acid secretion.

The nurse is conducting a community education program on peptic ulcer disease prevention. The nurse includes that the most common cause of peptic ulcers is: - stress and anxiety. - gram-negative bacteria. - alcohol and tobacco. - ibuprofen and aspirin.

Correct response: gram-negative bacteria. Explanation: The nurse should include that the most common cause of peptic ulcers is gram-negative bacteria (Helicobacter pylori).

The Zollinger-Ellison syndrome (ZES) consists of severe peptic ulcers, extreme gastric hyperacidity, and gastrin-secreting benign or malignant tumors of the pancreas. The nurse recognizes that an agent that is used to decrease bleeding and decrease gastric acid secretions is - nizatidine (Axid) - omeprazole (Prilosec) - vasopressin (Pitressin) - octreotide (Sandostatin)

Correct response: octreotide (Sandostatin) Explanation: For patients with ZES, hypersecretion of acid may be controlled with high doses of H2 receptor antagonists. These clients may require twice the normal dose, and dosages usually need to be increased with prolonged use. Octreotide (Sandostatin), a medication that suppresses gastrin levels, also may be prescribed.

A client with gastric cancer is having a resection. What is the nursing management priority for this client?

Correcting nutritional deficits

The nurse is providing preoperative care for a client with gastric cancer who is having a resection. What is the nursing management priority for this client? Correcting nutritional deficits Discharge planning Preventing deep vein thrombosis (DVT) Teaching about radiation treatment

Correcting nutritional deficits Explanation: Clients with gastric cancer commonly have nutritional deficits and may have cachexia. Therefore, correcting nutritional deficits is a top priority. Discharge planning before surgery is important, but correcting the nutritional deficits is a higher priority. Radiation therapy hasn't been proven effective for gastric cancer, and teaching about it preoperatively wouldn't be appropriate. Preventing DVT isn't a high priority before surgery, but it assumes greater importance after surgery.

Which ulcer is associated with extensive burn injury? Peptic ulcer Duodenal ulcer Curling ulcer Cushing ulcer

Curling ulcer Explanation: Curling ulcer is frequently observed about 72 hours after extensive burns and involves the antrum of the stomach or the duodenum.

A nurse caring for a patient in a burn treatment center knows to assess for the presence of which of the following types of ulcer about 72 hours post injury?

Curling's

A nurse caring for a patient in a burn treatment center knows to assess for the presence of which of the following types of ulcer about 72 hours post injury?

Curling's Curling's ulcer results from a complication from severe burns that causes reduced plasma volume that affects the gastric mucosa. Cushing's ulcer is produced by elevated intracranial pressure and is common with head injuries and brain trauma.

A client experienced extensive burns and 72 hours later has developed an ulcer. Which of the following types of ulcer is most likely in this client?

Curling's ulcer

A client sustained second- and third-degree burns over 30% of the body surface area approximately 72 hours ago. What type of ulcer should the nurse be alert for while caring for this client?

Curling's ulcer

A client sustained second- and third-degree burns over 30% of the body surface area approximately 72 hours ago. What type of ulcer should the nurse be alert for while caring for this client? Esophageal ulcer Meckel's ulcer Peptic ulcer Curling's ulcer

Curling's ulcer Explanation: Curling's ulcer is frequently observed about 72 hours after extensive burns and involves the antrum of the stomach or the duodenum. Peptic, esophageal, and Meckel's ulcers are not related to burn injuries.

A patient is complaining of diarrhea after having bariatric surgery. What nonpharmacologic treatment can the nurse suggest to decrease the incidence of diarrhea?

Decrease the fat content in the diet.

The nurse is caring for a client receiving enteral nutrition with a standard polymeric formula. For which reason will the nurse question using this formula for the client?

Diagnosed with malabsorption syndrome

Symptoms associated with pyloric obstruction include all of the following except:

Diarrhea

Which of the following appears to be a significant factor in the development of gastric cancer?

Diet

Which of the following appears to be a significant factor in the development of gastric cancer?

Diet Diet seems to be a significant factor: a diet high in smoked, salted, or pickled foods and low in fruits and vegetables may increase the risk of gastric cancer. The typical patient with gastric cancer is between 50 and 70 years of age. Men have a higher incidence than women. Native Americans, Hispanic Americans, and African Americans are twice as likely as Caucasian Americans to develop gastric cancer.

Which of the following appears to be a significant factor in the development of gastric cancer? Ethnicity Diet Age Gender

Diet Explanation: Diet seems to be a significant factor: a diet high in smoked, salted, or pickled foods and low in fruits and vegetables may increase the risk of gastric cancer. The typical patient with gastric cancer is between 50 and 70 years of age. Men have a higher incidence than women. Native Americans, Hispanic Americans, and African Americans are twice as likely as European Americans to develop gastric cancer.

A client with active schizophrenia has developed acute gastritis after ingesting a strongly alkaline solution during a psychotic episode. Corrosion is extensive. Which of the following emergency treatments might the team working with this client use? Select all that apply.

Diluted lemon juice Diluted vinegar

Inflammation of a diverticulum resulting in abscess formation Diverticulitis Diverticulosis Mechanical bowel obstruction Nonmechanical (functional, paralytic, ileus) bowel obstruction

Diverticulitis

Presence of several diverticula in the intestine Diverticulitis Diverticulosis Mechanical bowel obstruction Nonmechanical (functional, paralytic, ileus) bowel obstruction

Diverticulosis

The nurse reviews dietary guidelines with a client who had a gastric banding. Which teaching points are included? Select all that apply.

Do not eat and drink at the same time. Drink plenty of water, from 90 minutes after each meal to 15 minutes before each meal Avoid fruit drinks and soda.

The nurse reviews dietary guidelines with a client who had a gastric banding. Which teaching points are included? Select all that apply. Avoid fruit drinks and soda. Drink plenty of water, from 90 minutes after each meal to 15 minutes before each meal. Do not eat and drink at the same time. Limit meal size to 450 to 500 mL. Eat six meals a day.

Do not eat and drink at the same time. Drink plenty of water, from 90 minutes after each meal to 15 minutes before each meal. Avoid fruit drinks and soda. Explanation: Total meal size should be restricted to less than 8 oz or 240 mL. Three meals a day are recommended.

A client receiving tube feedings to the duodenum develops nausea, cramping, and diarrhea. For which condition should the nurse plan care for this client?

Dumping syndrome

A client who had a Roux-en-Y bypass procedure for morbid obesity ate a chocolate chip cookie after a meal. After ingestion of the cookie, the client reported cramping pains, dizziness, and palpitation. After having a bowel movement, the symptoms resolved. What should the nurse educate the client about regarding this event?

Dumping syndrome

A client who had a Roux-en-Y bypass procedure for morbid obesity ate a chocolate chip cookie after a meal. After ingestion of the cookie, the client reported cramping pains, dizziness, and palpitation. After having a bowel movement, the symptoms resolved. What should the nurse educate the client about regarding this event? Dumping syndrome Bile reflux Celiac disease Gastric outlet obstruction

Dumping syndrome Explanation: Dumping syndrome is an unpleasant set of vasomotor and GI symptoms that occur in up to 76% of patients who have had bariatric surgery. Early symptoms include a sensation of fullness, weakness, faintness, dizziness, palpitations, diaphoresis, cramping pains, and diarrhea. These symptoms resolve once the intestine has been evacuated (i.e., with defecation).

Clients with Type O blood are at higher risk for which of the following GI disorders?

Duodenal ulcers

Clients with Type O blood are at higher risk for which of the following GI disorders?

Duodenal ulcers Familial tendency also may be a significant predisposing factor. People with blood type O are more susceptible to peptic ulcers than are those with blood type A, B, or AB. Blood type is not a predisposing factor for gastric cancer, esophageal varices, and diverticulitis.

Clients with Type O blood are at higher risk for which of the following GI disorders? Diverticulitis Gastric cancer Duodenal ulcers Esophageal varices

Duodenal ulcers Explanation: Familial tendency also may be a significant predisposing factor. People with blood type O are more susceptible to peptic ulcers than are those with blood type A, B, or AB. Blood type is not a predisposing factor for gastric cancer, esophageal varices, and diverticulitis.

Which of the following is the first portion of the small intestine?

Duodenum The duodenum is the first portion of the small intestine, between the stomach and the jejunum. The pylorus is the opening between the stomach and duodenum. The peritoneum is the thin membrane that lines the inside of the wall of the abdomen and covers all the abdominal organs. The omentum is the fold of the peritoneum that surrounds the stomach and other organs of the abdomen

The nursing student approaches his instructor to discuss the plan of care for his client diagnosed with peptic ulcer disease. The student asks what is the most common site for peptic ulcer formation? The instructor would state which one of the following? Stomach Duodenum Esophagus Pylorus

Duodenum Explanation: Peptic ulcers occur mainly in the gastroduodenal mucosa because this tissue cannot withstand the digestive action of gastric acid (HCl) and pepsin.

The nurse is obtaining a history on a patient who comes to the clinic. What symptom described by the patient is one of the first symptoms associated with esophageal disease?

Dysphagia

The nurse is caring for a client who has developed dumping syndrome while recovering from a gastrectomy. What recommendation should the nurse make to the client?

Eat several small meals daily spaced at equal intervals.

The nurse is caring for a client who has developed dumping syndrome while recovering from a gastrectomy. What recommendation should the nurse make to the client? Drink a minimum of 12 ounces of fluid with each meal. Eat several small meals daily spaced at equal intervals. Sit upright when eating and for 30 minutes afterward. Choose foods that are high in simple carbohydrates.

Eat several small meals daily spaced at equal intervals.

The client is experiencing painful oral lesions following radiation for oropharyngeal cancer. Which instruction should the nurse give this client?

Eat soft or liquid foods.

A health care provider suspects that a client has peptic ulcer disease. With which diagnostic procedure would the nurse most likely prepare to assist?

Endoscopy

A physician suspects that a client has peptic ulcer disease. With which of the following diagnostic procedures would the nurse most likely prepare to assist?

Endoscopy

A health care provider suspects that a client has peptic ulcer disease. With which diagnostic procedure would the nurse most likely prepare to assist? Stool antigen test Endoscopy Barium study of the upper gastrointestinal tract Gastric secretion study

Endoscopy Explanation: Barium study of the upper GI tract may show an ulcer; however, endoscopy is the preferred diagnostic procedure because it allows direct visualization of inflammatory changes, ulcers, and lesions. Through endoscopy, a biopsy of the gastric mucosa and of any suspicious lesions can be obtained. Endoscopy may reveal lesions that, because of their size or location, are not evident on x-ray studies. Less invasive diagnostic measures for detecting H. pylori include serologic testing for antibodies against the H. pylori antigen, stool antigen test, and urea breath test.

A nurse is caring for a client who just has been diagnosed with a peptic ulcer. When teaching the client about his new diagnosis, how should the nurse best describe it?

Erosion of the lining of the stomach or intestine

The nurse is assessing a client with an ulcer for signs and symptoms of hemorrhage. The nurse interprets which condition as a sign/symptom of possible hemorrhage? Polyuria Hypertension Bradycardia Hematemesis

Hematemesis Explanation: The nurse interprets hematemesis as a sign/symptom of possible hemorrhage from the ulcer. Other signs that can indicate hemorrhage include tachycardia, hypotension, and oliguria/anuria.

A client with severe peptic ulcer disease has undergone surgery and is several hours postoperative. During assessment, the nurse notes that the client has developed cool skin, tachycardia, and labored breathing; the client also appears to be confused. Which of the following complications has the client most likely developed?

Hemorrhage

A client with severe peptic ulcer disease has undergone surgery and is several hours postoperative. During assessment, the nurse notes that the client has developed cool skin, tachycardia, labored breathing, and appears to be confused. Which complication has the client most likely developed?

Hemorrhage

A client with severe peptic ulcer disease has undergone surgery and is several hours postoperative. During assessment, the nurse notes that the client has developed cool skin, tachycardia, and labored breathing; the client also appears to be confused. Which of the following complications has the client most likely developed?

Hemorrhage Signs of hemorrhage following surgery include cool skin, confusion, increased heart rate, labored breathing, and blood in the stool. Signs of penetration and perforation are severe abdominal pain, rigid and tender abdomen, vomiting, elevated temperature, and increased heart rate. Indicators of pyloric obstruction are nausea, vomiting, distended abdomen, and abdominal pain.

The nurse is cautiously assessing a client admitted with peptic ulcer disease because the most common complication that occurs in 10% to 20% of clients is: Perforation Hemorrhage Pyloric obstruction Intractable ulcer

Hemorrhage Explanation: Hemorrhage, the most common complication, occurs in 10% to 20% of clients with peptic ulcers. Bleeding may be manifested by hematemesis or melena. Perforation is erosion of the ulcer through the gastric serosa into the peritoneal cavity without warning. Intractable ulcer refers to one that is hard to treat, relieve, or cure. Pyloric obstruction, also called gastric outlet obstruction (GOO), occurs when the area distal to the pyloric sphincter becomes scarred and stenosed from spasm or edema or from scar tissue that forms when an ulcer alternately heals and breaks down.

A client with severe peptic ulcer disease has undergone surgery and is several hours postoperative. During assessment, the nurse notes that the client has developed cool skin, tachycardia, labored breathing, and appears to be confused. Which complication has the client most likely developed? Penetration Perforation Hemorrhage Pyloric obstruction

Hemorrhage Explanation: Signs of hemorrhage following surgery include cool skin, confusion, increased heart rate, labored breathing, and blood in the stool. Signs of penetration and perforation are severe abdominal pain, rigid and tender abdomen, vomiting, elevated temperature, and increased heart rate. Indicators of pyloric obstruction are nausea, vomiting, distended abdomen, and abdominal pain.

The nurse is caring for a client who has just returned from the PACU after surgery for peptic ulcer disease. For what potential complications does the nurse know to monitor? Select all that apply.

Hemorrhage Perforation Penetration Pyloric obstruction

The nurse is caring for a client who has just returned from the PACU after surgery for peptic ulcer disease. For what potential complications does the nurse know to monitor? Select all that apply

Hemorrhage Perforation Penetration Pyloric obstruction Potential complications may include hemorrhage, perforation, penetration, and pyloric obstruction. A client who has had surgery for peptic ulcer disease may have a decreased appetite in the immediate postoperative stage, but it is not something the nurse would monitor for and would not cause cachexia. Inability to clear secretions is generally not a complication of peptic ulcer surgery.

A client asks the nursing assistant for a bedpan. When the client is finished, the nursing assistant notifies the nurse that the client has bright red streaking of blood in the stool. The nurse's assessment should focus on what potential cause? Diet high in red meat Upper GI bleed Hemorrhoids Use of iron supplements

Hemorrhoids

A nurse is performing health education with a client who has a history of frequent, serious dental caries. When planning educational interventions, the nurse should identify a risk for what nursing diagnosis?

Imbalanced Nutrition: Less Than Body Requirements

During assessment of a patient with gastritis, the nurse practitioner attempts to distinguish acute from chronic pathology. One criteria, characteristic of gastritis would be the:

Immediacy of the occurrence.

A client diagnosed with a peptic ulcer says, "Now I have something else I have to worry about." Which actions will the nurse take to help reduce the client's anxiety? Select all that apply. Inform the client the medication will solve the problem. Interact with the client in a relaxed manner. Help identify the client's current stressors. Discuss potential coping techniques with the client. Offer information about relaxation methods.

Interact with the client in a relaxed manner. Help identify the client's current stressors. Discuss potential coping techniques with the client. Offer information about relaxation methods. Explanation: A client with a peptic ulcer may have a problem with anxiety. To help reduce the client's anxiety, the nurse should interact with the client in a relaxed manner and help the client identify stressors. The nurse can also discuss potential coping techniques and offer information about relaxation methods. Stating that medication will solve the problem may not be sufficient if stress and anxiety are contributors to the development of the ulcer

A nursing student is caring for a client with gastritis. Which of the following would the student recognize as a common cause of gastritis? Choose all that apply.

Irritating foods Ingestion of strong acids Overuse of aspirin Acute gastritis is often caused by dietary indiscretion—a person eats food that is irritating, too highly seasoned, or contaminated with disease-causing microorganisms. Other causes of acute gastritis include overuse of aspirin and other nonsteroidal anti-inflammatory drugs (NSAIDs), excessive alcohol intake, bile reflux, and radiation therapy. A more severe form of acute gastritis is caused by the ingestion of strong acid or alkali, which may cause the mucosa to become gangrenous or to perforate. A DASH diet is an accronym for Dietary Approaches to Stop Hypertension, which would not cause gastritis. Participation in competitive sports also would not cause gastritis.

Which is a true statement regarding the nursing considerations in administration of metronidazole?

It leaves a metallic taste in the mouth.

Which of the following is a true statement regarding the nursing considerations in administration of metronidazole (Flagyl)?

It leaves a metallic taste in the mouth. Flagyl leaves a metallic taste in the mouth. It may cause anorexia and should be given with meals to decrease GI upset. Flagyl increases the blood-thinning effects of warfarin (Coumadin).

A nurse is providing client education for a client with peptic ulcer disease secondary to chronic nonsteroidal anti-inflammatory drug (NSAID) use. The client has recently been prescribed misoprostol. What would the nurse be most accurate in informing the client about the drug? It reduces the stomach's volume of hydrochloric acid It increases the speed of gastric emptying It protects the stomach's lining It increases lower esophageal sphincter pressure

It protects the stomach's lining

A 66-year-old African-American client has recently visited a physician to confirm a diagnosis of gastric cancer. The client has a history of tobacco use and was diagnosed 10 years ago with pernicious anemia. He and his family are shocked about the possibility of cancer because he was asymptomatic prior to recent complaints of pain and multiple gastrointestinal symptoms. On the basis of knowledge of disease progression, the nurse assumes that organs adjacent to the stomach are also affected. Which of the following organs may be affected? Choose all that apply.

Liver Pancreas Duodenum

A 66-year-old African-American client has recently visited a physician to confirm a diagnosis of gastric cancer. The client has a history of tobacco use and was diagnosed 10 years ago with pernicious anemia. He and his family are shocked about the possibility of cancer because he was asymptomatic prior to recent complaints of pain and multiple gastrointestinal symptoms. On the basis of knowledge of disease progression, the nurse assumes that organs adjacent to the stomach are also affected. Which of the following organs may be affected? Choose all that apply. Duodenum Bladder Pancreas Liver Lungs

Liver Pancreas Duodenum Explanation: Most gastric cancers are adenocarcinomas; they can occur anywhere in the stomach. The tumor infiltrates the surrounding mucosa, penetrating the wall of the stomach and adjacent organs and structures. The liver, pancreas, esophagus, and duodenum are often already affected at the time of diagnosis. Metastasis through lymph to the peritoneal cavity occurs later in the disease.

A client is recovering in the hospital following gastrectomy. The nurse notes that the client has become increasingly difficult to engage and has had several angry outbursts at staff members in recent days. The nurse's attempts at therapeutic dialogue have been rebuffed. What is the nurse's most appropriate action?

Make appropriate referrals to services that provide psychosocial support.

The nurse visits the home of a client recovering from acute gastritis. Which observation indicates that teaching about the disorder was effective?

Medications placed in a pillbox

Rebleeding may occur from a peptic ulcer and often warrants surgical interventions. Signs of bleeding include which of the following?

Mental confusion Signs of bleeding include tachycardia, tachypnea, hypotension, mental confusion, thirst, and oliguria.

Rebleeding may occur from a peptic ulcer and often warrants surgical interventions. Signs of bleeding include which of the following? Hypertension Mental confusion Bradypnea Bradycardia

Mental confusion Explanation: Signs of bleeding include tachycardia, tachypnea, hypotension, mental confusion, thirst, and oliguria.

A client who underwent a gastric resection 3 weeks ago is having their diet progressed on a daily basis. Following the latest meal, the client reports dizziness and palpitations. Inspection reveals that the client is diaphoretic. What is the nurse's best action?

Monitor the client closely for further signs of dumping syndrome.

Which of the following is a true statement regarding gastric cancer?

Most patients are asymptomatic during the early stage of the disease. Most patients are asymptomatic during the early stage of the disease. Men have a higher incidence of gastric cancer. The prognosis is poor because the diagnosis is usually made late because most patients are asymptomatic during the early stage. Most cases of gastric cancer are discovered only after local invasion has advanced or metastases are present.

The nurse in the ED admits a client with suspected gastric outlet obstruction. The client's symptoms include nausea and vomiting. The nurse anticipates that the physician will issue which order?

Nasogastric tube insertion

The nurse in the ED admits a client with suspected gastric outlet obstruction. The client's symptoms include nausea and vomiting. The nurse anticipates that the physician will issue which order? Stool specimen Nasogastric tube insertion Oral contrast Pelvic x-ray

Nasogastric tube insertion Explanation: The nurse anticipates an order for nasogastric tube insertion to decompress the stomach. Pelvic x-ray, oral contrast, and stool specimens are not indicated at this time.

A 75-year-old male patient presents at the emergency department with symptoms of a small bowel obstruction. An emergency room nurse is obtaining assessment data from this patient. What assessment finding is characteristic of a small bowel obstruction? Mucosal edema Mucus in stool Nausea and vomiting Decrease in urine production

Nausea and vomiting

A nurse is caring for a client with active upper GI bleeding. What is the appropriate diet for this client during the first 24 hours after admission?

Nothing by mouth

A nurse is caring for a client with active upper GI bleeding. What is the appropriate diet for this client during the first 24 hours after admission?

Nothing by mouth Shock and bleeding must be controlled before oral intake, so the client should receive nothing by mouth. When the bleeding is controlled, the diet is gradually increased, starting with ice chips and then clear liquids. Skim milk shouldn't be given because it increases gastric acid production, which could prolong bleeding. A clear liquid diet is the first diet offered after bleeding and shock are controlled.

A client undergoes total gastrectomy. Several hours after surgery, the nurse notes that the client's nasogastric (NG) tube has stopped draining. How should the nurse respond?

Notify the health care provider.

The nurse working in the recovery room is caring for a client who had a radical neck dissection. The nurse notices that the client makes a coarse, high-pitched sound upon inspiration. Which intervention by the nurse is appropriate?

Notify the physician

During a home visit the nurse notes that a client recovering from peptic ulcer disease is experiencing cool clammy skin and has a heart rate of 96 beats a minute. Which action will the nurse take?

Notify the primary health care provider.

During a home visit the nurse notes that a client recovering from peptic ulcer disease is experiencing cool clammy skin and has a heart rate of 96 beats a minute. Which action will the nurse take? Discuss the types of foods the client has been eating. Notify the primary health care provider. Provide a dose of a proton pump inhibitor. Encourage the client to drink a warm beverage.

Notify the primary health care provider. Explanation: The client with peptic ulcer disease is demonstrating signs of hemorrhage which include cool skin and tachycardia. The health care provider should be immediately notified. The client should not be given any additional medication. A warm beverage could enhance bleeding. It is inappropriate to provide any teaching while the client is experiencing an acute condition.

The nurse is conducting a community health education program on obesity. The nurse includes which of the following diseases/disorders in the program?

Obstructive sleep apnea The nurse includes that obstructive sleep apnea is a disease/disorder associated with obesity as well as asthma; breast, endometrial, prostate, renal, colon, and gallbladder cancer; osteoarthritis, coronary artery disease, cholecystitis, cholelithiasis, chronic back pain, diabetes, hypertension, coronary artery disease, heart failure, and pulmonary embolism.

A nurse practitioner prescribes drug therapy for a patient with peptic ulcer disease. Choose the drug that can be used for 4 weeks and has a 90% chance of healing the ulcer.

Omeprazole

Which medication classification represents a proton (gastric acid) pump inhibitor?

Omeprazole

A nurse practitioner prescribes drug therapy for a patient with peptic ulcer disease. Choose the drug that can be used for 4 weeks and has a 90% chance of healing the ulcer.

Omeprazole (Prilosec)

The nurse is conducting a community education class on gastritis. The nurse includes that chronic gastritis caused by Helicobacter pylori is implicated in which of the following diseases/conditions?

Peptic ulcers Chronic gastritis caused by Helicobacter pylori is implicated in the development of peptic ulcers. Chronic gastritis is sometimes associated with autoimmune disease, such as pernicious anemia, but not as a cause of the anemia. Chronic gastritis is not implicated in system infections and/or colostomies.

A client is in the hospital for the treatment of peptic ulcer disease. The client reports vomiting and a sudden severe pain in the abdomen. The nurse then assesses a board-like abdomen. What does the nurse suspect these symptoms indicate?

Perforation of the peptic ulcer

A client is in the hospital for the treatment of peptic ulcer disease. The client reports vomiting and a sudden severe pain in the abdomen. The nurse then assesses a board-like abdomen. What does the nurse suspect these symptoms indicate? Gastric penetration Ineffective treatment for the peptic ulcer A reaction to the medication given for the ulcer Perforation of the peptic ulcer

Perforation of the peptic ulcer Explanation: Signs and symptoms of perforation include the following: Sudden, severe upper abdominal pain (persisting and increasing in intensity), which may be referred to the shoulders, especially the right shoulder, because of irritation of the phrenic nerve in the diaphragm; vomiting; collapse (fainting); extremely tender and rigid (board-like) abdomen; and hypotension and tachycardia, indicating shock.

The nurse is caring for a patient who is suspected to have developed a peptic ulcer hemorrhage. Which action would the nurse perform first?

Place the patient in a recumbent position with the legs elevated.

The nurse's comprehensive assessment of a client includes inspection for signs of oral cancer. What assessment finding is most characteristic of oral cancer in its early stages?

Presence of a painless sore with raised edges

The nurse is preparing to check for tube placement in the client's stomach as well as measure the residual volume. What is the main purpose of these nursing actions?

Prevent aspiration

The nurse is teaching a client with peptic ulcer disease who has been prescribed misoprostol. What information from the nurse would be most accurate about misoprostol?

Prevents ulceration in clients taking nonsteroidal anti-inflammatory drugs (NSAIDs)

The nurse is teaching a client with peptic ulcer disease who has been prescribed misoprostol. What information from the nurse would be most accurate about misoprostol? Decreases mucus production Prevents ulceration in clients taking nonsteroidal anti-inflammatory drugs (NSAIDs) Works best when taken on an empty stomach Increases the speed of gastric emptying

Prevents ulceration in clients taking nonsteroidal anti-inflammatory drugs (NSAIDs) Explanation: Misoprostol (Cytotec) is a synthetic prostaglandin that protects the gastric mucosa against ulceration and is used in clients who take NSAIDs. Misoprostol should be taken with food. It does not improve emptying of the stomach, and it increases (not decreases) mucus production.

A nurse is preparing to discharge a client newly diagnosed with peptic ulcer disease. The client's diagnostic test results were positive for H. pylori bacteria. The health care provider has ordered the "triple therapy" regimen. Which medications will the nurse educate the client on? H2-receptor antagonist and two antibiotics Proton-pump inhibitor, an antibiotic, and bismuth salts H2-receptor antagonist, proton-pump inhibitor, and an antibiotic Proton-pump inhibitor and two antibiotics

Proton-pump inhibitor and two antibiotics Explanation: Currently, the most commonly used therapy for peptic ulcers is a combination of antibiotics, proton-pump inhibitors, and bismuth salts that suppress or eradicate H. pylori bacteria. Recommended therapy for 10 to 14 days includes triple therapy with two antibiotics (e.g., metronidazole [Flagyl] or amoxicillin [Amoxil] and clarithromycin [Biaxin]) plus a proton-pump inhibitor (e.g., lansoprazole [Prevacid], omeprazole [Prilosec], or rabeprazole [Aciphex]), or quadruple therapy with two antibiotics (metronidazole and tetracycline) plus a proton-pump inhibitor and bismuth salts (Pepto-Bismol). Research is being conducted to develop a vaccine against H. pylori.

A patient has been diagnosed with a hiatal hernia. The nurse explains the diagnosis to the patient and his family by telling them that a hernia is a (an):

Protrusion of the upper stomach into the lower portion of the thorax.

The nurse is caring for a client with chronic gastritis. Which interventions will the nurse add to this client's plan of care? Select all that apply. Suggest using over the counter ibuprofen for pain control. Instruct to avoid foods that aggravate the condition. Review actions to reduce stress. Provide omeprazole as prescribed. Remind to avoid alcohol intake.

Remind to avoid alcohol intake. Review actions to reduce stress. Provide omeprazole as prescribed. Instruct to avoid foods that aggravate the condition. Explanation: Chronic gastritis is managed by recommending avoidance of alcohol, reducing stress, providing medications to include proton pump inhibitors, and modifying the diet by avoiding foods that aggravate the condition. Over-the-counter NSAIDs should be avoided as this can aggravate the condition.

A client recovering from surgery to resect a gastric tumor reports abdominal cramping and diarrhea after eating. Which additional symptoms will the nurse use to determine if the client is experiencing dumping syndrome? Select all that apply. Inability to tolerate foods with animal fat Audible bowel sounds and pain radiating to the back Increased desire to consume foods that are dairy products Sudden onset of extreme shakiness and fatigue Report of extreme hunger 3 hours after eating

Report of extreme hunger 3 hours after eating Sudden onset of extreme shakiness and fatigue Explanation: Dumping syndrome may occur as a result of any surgical procedure that involves the removal of a significant portion of the stomach. Early symptoms tend to occur within 10 to 30 minutes after a meal and often include early satiety, cramping abdominal pain, nausea, vomiting, and diarrhea. Later, the rapid elevation in blood glucose is followed by the increased secretion of insulin, which results in hypoglycemia 2 to 3 hours after eating. Manifestations of hypoglycemia may include extreme hunger, shakiness, and fatigue. Findings associated with dumping syndrome do not include an intolerance of foods with animal fat, audible bowel sounds with pain radiating to the back, or a desire to consume dairy products.

Which of the following is included as a dietary guideline for a patient who has had bariatric surgery?

Restrict total meal size to less than 1 cup. Dietary guidelines would include a restriction of total meal size to less than 1 cup. The patient should eat three meals per day. Avoid liquid calories such as soda. The patient should drink plenty of water, from 90 minutes after each meal to 15 minutes before the next meal.

Which of the following are characteristics associated with the Zollinger-Ellison syndrome (ZES)? Select all that apply. Gastrin-secreting tumors of the pancreas Severe peptic ulcers Extreme gastric hyperacidity Hypocalcemia Constipation

Severe peptic ulcers Extreme gastric hyperacidity Gastrin-secreting tumors of the pancreas Explanation: ZES consists of severe peptic ulcers, extreme gastric hyperacidity, and gastrin-secreting benign or malignant tumors of the pancreas. Diarrhea and steatorrhea may be evident. The client may have co-existing parathyroid adenomas or hyperplasia and may therefore exhibit signs of hypercalcemia.

A patient has a Class II classification of obesity. What level of health risk does this pose for the patient?

Severe risk Class II obesity is associated with severe risk. Overweight is associated with mild risk, Class I obesity with moderate risk, and Class III obesity with very severe risk.

A client is preparing for discharge to home following a partial gastrectomy and vagotomy. Which is the best rationale for the client being taught to lie down for 30 minutes after each meal? Slows gastric emptying Provides much needed rest Removes tension on internal suture line Allows for better absorption of vitamin B12

Slows gastric emptying Explanation: Dumping syndrome is a common complication following subtotal gastrectomy. To avoid the rapid emptying of stomach contents, resting after meals can be helpful. Promoting rest after a major surgery is helpful in recovery but not the reason for resting after meals. Following this type of surgery, clients will have a need for vitamin B12 supplementation due to absence of production of intrinsic factor in the stomach. Resting does not increase absorption of B12 or remove tension on suture line.

A client with peptic ulcer disease wants to know nonpharmacologic ways that he can prevent recurrence. Which of the following measures would the nurse recommend? Select all that apply.

Smoking cessation Avoidance of alcohol Following a regular schedule for rest, relaxation, and meals

A nurse is caring for a client hospitalized with an exacerbation of chronic gastritis. What health promotion topic should the nurse emphasize?

Strategies for avoiding irritating foods and beverages

Which of the following are signs/symptoms of perforation?

Sudden, severe upper abdominal pain

A client has sought care because of recent dark-colored stools. As a result, a fecal occult blood test has been ordered. The nurse should give what instructions to the client? Take no NSAIDs within 48 hours of the test Take prescribed medications as usual Avoid over the counter vitamin C supplements Do not use fiber supplements before the test

Take no NSAIDs within 48 hours of the test

A client with peptic ulcer disease has been prescribed sucralfate. What health education should the nurse provide to this client?

Take the medication 2 hours before or after other medications

A client with peptic ulcer disease has been prescribed sucralfate. What health education should the nurse provide to this client? Take the medication 2 hours before or after other medications Blood levels will be evaluated after 1 week Take the medication at bedtime to accommodate sedative effects Ensure adequate potassium intake during therapy

Take the medication 2 hours before or after other medications

A client taking metronidazole for the treatment of H. pylori states that the medication is causing nausea. What teaching should the nurse provide to the client to alleviate the nausea?

Take the medication with meals to decrease the nausea.

A healthcare provider prescribes a combination of drugs to treat reoccurring peptic ulcer disease, and the client asks the nurse the reason for all the medications. What teaching should the nurse review with the client? The proton pump inhibitors, prostaglandin E1 analogs, and bismuth salts will suppress or eradicate H. pylori. The antibiotics, prostaglandin E1 analogs, and bismuth salts will work together to suppress or eradicate H. pylori. The prostaglandin E1 analogs, antibiotics, and proton pump inhibitors will work together to suppress or eradicate H. pylori. The bismuth salts, antibiotics, and proton pump inhibitors will work together to suppress or eradicate H. pylori.

The bismuth salts, antibiotics, and proton pump inhibitors will work together to suppress or eradicate H. pylori. Explanation: The recommended combination of bismuth salts, antibiotics, and proton pump inhibitors will suppress or eradicate H. pylori. Prostaglandin E1 analogs enhance mucosal resistance to injury; they do not suppress or eradicate H. pylori.

Which of the following is a true statement regarding regional enteritis (Crohn's disease)? It has a progressive disease pattern It is characterized by lower left quadrant abdominal pain. The clusters of ulcers take on a cobblestone appearance. The lesions are in continous contact with one another.

The clusters of ulcers take on a cobblestone appearance.

A nurse is admitting a client diagnosed with late-stage gastric cancer. The client's family is distraught and angry that the client was not diagnosed earlier in the course of her disease. What factor most likely contributed to the client's late diagnosis?

The early symptoms of gastric cancer are usually not alarming or highly unusual.

The nurse is preparing a teaching tool about delayed release proton pump inhibitors used to treat duodenal ulcer disease caused by H. pylori. Which statement will the nurse include that would apply to most types of proton pump inhibitor prescribed to treat this condition? May cause diarrhea and hyperglycemia Abdominal pain and abnormal liver function tests are expected effects Interferes with the metabolism of digoxin, iron, and warfarin The medication is to be swallowed whole and taken before meals

The medication is to be swallowed whole and taken before meals Explanation: There are several proton pump inhibitors used to treat duodenal ulcers caused by H. pylori. For most of these medications, they are a delayed-release capsule that is to be swallowed whole and taken before meals. Pantoprazole may cause diarrhea and hyperglycemia. Rabeprazole is the only proton pump inhibitor that interferes with the metabolism of digoxin, iron, and warfarin. Rabeprazole may cause abdominal pain. Pantoprazole may cause abnormal liver function tests.

Which of the following interventions are appropriate for clients with gastritis? Select all that apply. Discourage cigarette smoking. Provide general education about how to prevent recurrences. Notify the physician of indicators of hemorrhagic gastritis. Give the client food and fluids every 4 hours. Use a calm approach to reduce anxiety.

Use a calm approach to reduce anxiety. Discourage cigarette smoking. Notify the physician of indicators of hemorrhagic gastritis. Provide general education about how to prevent recurrences. Explanation: The nurse should use a calm approach when answering questions and providing teaching. He or she should discuss smoking cessation and monitor for any indicators of hemorrhagic gastritis. The client will take nothing by mouth for up to a few days until symptoms subside. The nurse needs to develop an individualized teaching plan for the client that includes information about stress management, diet, and medications.

A nurse is providing care for a client recovering from gastric bypass surgery. During assessment, the client exhibits pallor, perspiration, palpitations, headache, and feelings of warmth, dizziness, and drowsiness. The client reports eating 90 minutes ago. What will the nurse suspect?

Vasomotor symptoms associated with dumping syndrome

A nurse is providing care for a client recovering from gastric bypass surgery. During assessment, the client exhibits pallor, perspiration, palpitations, headache, and feelings of warmth, dizziness, and drowsiness. The client reports eating 90 minutes ago. The nurse suspects:

Vasomotor symptoms associated with dumping syndrome Early manifestations of dumping syndrome occur 15 to 30 minutes after eating. Signs and symptoms include vertigo, tachycardia, syncope, sweating, pallor, palpitations, diarrhea, nausea, and the desire to lie down. Dehiscence of the surgical wound is characterized by pain and a pulling or popping feeling at the surgical site. Peritonitis presents with a rigid, boardlike abdomen, tenderness, and fever. The client's signs and symptoms aren't a normal reaction to surgery.

A nurse is teaching a group of middle-aged men about peptic ulcers. When discussing risk factors for peptic ulcers, the nurse should mention:

alcohol abuse and smoking. The nurse should mention that risk factors for peptic (gastric and duodenal) ulcers include alcohol abuse, smoking, and stress.

A nurse is teaching a group of middle-aged men about peptic ulcers. When discussing risk factors for peptic ulcers, the nurse should mention: a sedentary lifestyle and smoking. alcohol abuse and a history of acute renal failure. alcohol abuse and smoking. a history of hemorrhoids and smoking.

alcohol abuse and smoking. Explanation: The nurse should mention that risk factors for peptic (gastric and duodenal) ulcers include alcohol abuse, smoking, and stress. A sedentary lifestyle and a history of hemorrhoids aren't risk factors for peptic ulcers. Chronic renal failure, not acute renal failure, is associated with duodenal ulcers.

A client is admitted to the health care facility with a diagnosis of a bleeding gastric ulcer. The nurse expects this client's stools to be:

black and tarry.

A client is admitted to the health care facility with a diagnosis of a bleeding gastric ulcer. The nurse expects this client's stools to be:

black and tarry. Black, tarry stools are a sign of bleeding high in the GI tract, as from a gastric ulcer, and result from the action of digestive enzymes on the blood. Vomitus associated with upper GI tract bleeding commonly is described as coffee-ground-like. Clay-colored stools are associated with biliary obstruction. Bright red stools indicate lower GI tract bleeding.

The nurse is conducting a community education program on peptic ulcer disease prevention. The nurse includes that the most common cause of peptic ulcers is:

gram-negative bacteria.

A nurse has auscultated a client's abdomen and noted one or two bowel sounds in a 2-minute period of time. How should the nurse document the client's bowel sounds? normal hypoactive hyperactive paralytic ileus

hypoactive

The nurse recognizes that the patient diagnosed with a duodenal ulcer will likely experience

pain 2 to 3 hours after a meal. The patient with a gastric ulcer often awakens between 1 to 2 with pain, and ingestion of food brings relief

The nurse recognizes that the client diagnosed with a duodenal ulcer will likely experience hemorrhage. vomiting. weight loss. pain 2 to 3 hours after a meal.

pain 2 to 3 hours after a meal. Explanation: The client with a duodenal ulcer often awakens between 1 and 2 with pain, and ingestion of food brings relief. Vomiting is uncommon in the client with duodenal ulcer. Hemorrhage is less likely in the client with duodenal ulcer than in the client with gastric ulcer. The client with a duodenal ulcer may experience weight gain.


Set pelajaran terkait

Gyakori MAGYAR kifejezések ANGOLUL

View Set

Module 41(Anxiety disorders, ocd, and ptsd)

View Set

LS 7A--Week 6: Protein Structure & Translation

View Set

Bio 1620 - Exam 1 - Lecture quizzes

View Set

Chapter 6: Corporate Level Strategy: Creating Value through Diversification, Chapter 5: Business-Level Strategy: Creating and Sustaining Competitive Advantages, Ch. 7: International Strategy: Creating Value in Global Markets

View Set